Buy BOOKS at Discounted Price

Matrices

Class 12th Mathematics NCERT Exemplar Solution
Exercise
  1. If a matrix has 28 elements, what are the possible orders it can have? What if it…
  2. In the matrix a = [ccc a&1 2& root 3 & x^2 - y 0&5& - 2/5] , write: (i) The order…
  3. Construct a2×2 matrix where (i) a_ij = (i-2 j)/2 (ii) aij = |- 2i + 3j|…
  4. Construct a 3 × 2 matrix whose elements are given by aij = eixsin jx…
  5. Find values of a and b if A = B, where a = [cc a+4&3b 8&-6] and b = [cc 2a+2& b^2…
  6. If possible, find the sum of the matrices A and B, where a = [cc root 3 &1 2&3]…
  7. If x = [ccc 3&1&-1 5&-2&-3] and y = [ccc 2&1&-1 7&2&4] find (i) X + Y (ii) 2X - 3Y…
  8. Find non-zero values of x satisfying the matrix equation: x [cc 2x&2 3]+2 [cc 8&5x…
  9. If a = [ll 0&1 1&1] and b = [cc 0&-1 1&0] , show that (A + B) (A - B) ≠ A^2 - B^2…
  10. Find the value of x if [lll 1&1] [ccc 1&3&2 2&5&1 15&3&2] [1 2 x] = 0…
  11. Show that a = [cc 5&3 -1&-2] satisfies the equation A^2 - 3A - 7I = 0 and hence…
  12. Find the matrix A satisfying the matrix equation: [ll 2&1 3&2]a [cc -3&2 5&-3] =…
  13. Find A, if [4 1 3]a = [rrr -4&8&4 -1&2&1 -3&6&3] .
  14. If a = [cc 3&-4 1&1 2&0] and b = [lll 2&1&2 1&2&4] , then verify (BA)^2 ≠ B^2 A^2…
  15. If possible, find BA and AB, where a = [lll 2&1&2 1&2&4] , b = [ll 4&1 2&3 1&2]…
  16. Show by an example that for A ≠ O, B ≠ O, AB = O.
  17. Given a = [lll 2&4&0 3&9&6] and b = [ll 1&4 2&8 1&3] . Is (AB)’ = B’A’?…
  18. Solve for x and y: x [2 1]+y [3 5] + [-8 -11] = 0
  19. If X and Y are 2 × 2 matrices, then solve the following matrix equations for X…
  20. If A = [3 5], B = [7 3], then find a non-zero matrix C such that AC = BC.…
  21. Given an example of matrices A, B and C such that AB = AC, where A is non-zero…
  22. If a = [cc 1&2 -2&1] , b = [cc 2&3 3&-4] and c = [cc 1&0 -1&0] , verify: (i) (AB)…
  23. If p = [lll x&0&0 0&0 0&0] , q = [lll a&0&0 0&0 0&0] , prove that pq = [lll…
  24. If [ccc 2&1&3] [ccc -1&0&-1 -1&1&0 0&1&1] [c 1 0 -1] = a , find A.…
  25. If a = [ll 2&1] , b = [lll 5&3&4 8&7&6] and c = [ccc -1&2&1 1&0&2] , verify that…
  26. If a = [ccc 1&0&-1 2&1&3 0&1&1] , then verify that A^2 + A = A(A + I), where I is…
  27. If a = [ccc 0&-1&2 4&3&-4] and b = [ll 4&0 1&3 2&6] , then verify that: (i) (A’)’…
  28. If a = [ll 1&2 4&1 5&6] , b = [ll 1&2 6&4 7&3] then verify that: (i) (2A + B)’ =…
  29. Show that A’A and AA’ are both symmetric matrices for any matrix A.…
  30. Let A and B be square matrices of the order 3 × 3. Is (AB)^2 = A^2 B^2 ? Give…
  31. Show that if A and B are square matrices such that AB = BA, then (A + B)^2 = A^2…
  32. Let a = [cc 1&2 -1&3] , b = [ll 4&0 1&5] , c = [cc 2&0 1&-2] and a = 4, b = -2.…
  33. Let a = [cc 1&2 -1&3] , b = [ll 4&0 1&5] , c = [cc 2&0 1&-2] and a = 4, b = -2.…
  34. Let a = [cc 1&2 -1&3] , b = [ll 4&0 1&5] , c = [cc 2&0 1&-2] and a = 4, b = -2.…
  35. Let a = [cc 1&2 -1&3] , b = [ll 4&0 1&5] , c = [cc 2&0 1&-2] and a = 4, b = -2.…
  36. Let a = [cc 1&2 -1&3] , b = [ll 4&0 1&5] , c = [cc 2&0 1&-2] and a = 4, b = -2.…
  37. Let a = [cc 1&2 -1&3] , b = [ll 4&0 1&5] , c = [cc 2&0 1&-2] and a = 4, b = -2.…
  38. Let a = [cc 1&2 -1&3] , b = [ll 4&0 1&5] , c = [cc 2&0 1&-2] and a = 4, b = -2.…
  39. Let a = [cc 1&2 -1&3] , b = [ll 4&0 1&5] , c = [cc 2&0 1&-2] and a = 4, b = -2.…
  40. Let a = [cc 1&2 -1&3] , b = [ll 4&0 1&5] , c = [cc 2&0 1&-2] and a = 4, b = -2.…
  41. If [cc costheta heta -sintegrate heta] then show that a^2 = [cc cos2theta…
  42. If a = [cc 0&-x x&0] , b = [ll 0&1 1&0] and x^2 = -1, then show that (A + B)^2 =…
  43. Verify that A^2 = I when a = [ccc 0&1&-1 4&-3&4 3&-3&4]
  44. Prove by Mathematical Induction that (A’)n = (An)’, where n ∈ N for any square…
  45. [cc 1&3 -5&7] Find inverse, by elementary row operations (if possible), of the…
  46. [cc 1&-3 -2&6] Find inverse, by elementary row operations (if possible), of the…
  47. If [cc xy&4 z+6+y] = [cc 8 0&6] then find values of x, y, z and w.…
  48. If a = [ll 1&5 7&12] b = [ll 9&1 7&8] find a matrix C such that 3A + 5B + 2C is a…
  49. If a = [cc 3&-5 -4&2] then find A^2 - 5A - 14I. Hence, obtain A^3 .…
  50. Find the value of a, b, c and d, if 3 [cc a c] = [cc a&6 -1&2d] + [cc 4+b c+d&3]…
  51. Find the matrix A such that [rr 2&-1 1&0 -3&4]a = [ccc -1&-8&-10 1&-2&-5 9&22&15]…
  52. If a = [ll 1&2 4&1] find A^2 + 2A + 7I
  53. If a = [ll cosalpha -sinx] and A-1 = A’, find value of α.
  54. If the matrix [ccc 0&3 2&-1 c&1&0] is a skew symmetric matrix, find the values of…
  55. If p (x) = [cc cosx -sinx] then show that P(x).P(y) = P(x + y) = P(y).P(x)…
  56. If A is square matrix such that A^2 = A, show that (I + A)^3 = 7A + I.…
  57. If A, B are square matrices of same order and B is a skew-symmetric matrix, show…
  58. If AB = BA for any two square matrices, prove by mathematical induction that…
  59. Find x, y, z if a = [ccc 0&2y x&-z x&-y] satisfies A’ = A-1
  60. [ccc 2&-1&3 -5&3&1 -3&2&3] If possible, using elementary row transformations,…
  61. [ccc 2&3&-3 -1&-2&2 1&1&-1] If possible, using elementary row transformations,…
  62. [ccc 2&0&-1 5&1&0 0&1&3] If possible, using elementary row transformations, find…
  63. The matrix p = [lll 0&0&4 0&4&0 4&0&0] is aA. square matrix B. diagonal matrix C.…
  64. Total number of possible matrices of order 3 × 3 with each entry 2 or 0 isA. 9 B.…
  65. If [ll 2x+y&4x 5x-7&4x] = [ll 7&7y-13 y+6] then the value of x + y isA. x = 3, y…
  66. If a = 1/pi [cc sin^-1 (x pi) & tan^-1 x/pi sin^-1 x/pi & cos^-1 (pi x)] b = 1/pi…
  67. If A and B are two matrices of the order 3 × m and 3 × n, respectively, and m =…
  68. If a = [ll 0&1 1&0] then A^2 is equal toA. [ll 0&1 1&0] B. [ll 1&0 1&0] C. [ll…
  69. If matrix a = [a_ij]_ 2 x 2 , where aij = 1 if i ≠ j aij = 0 if i = j, then A^2…
  70. The matrix [lll 1&0&0 0&2&0 0&0&4] is aA. identity matrix B. symmetric matrix C.…
  71. The matrix [ccc 0&-5&8 5&0&12 -8&-12&0] is aA. diagonal matrix B. symmetric…
  72. If A is matrix of order m × n and B is a matrix such that AB’ and B’A are both…
  73. If A and B are matrices of same order, then (AB’ - BA’) is aA. skew symmetric…
  74. If A is a square matrix such that A^2 = I, then (A - I)^3 + (A + I)^3 - 7A is…
  75. For any two matrices A and B, we haveA. AB = BA B. AB ≠ BA C. AB = O D. None of…
  76. On using elementary column operations C2→ C2 — 2C1 in the following matrix…
  77. On using elementary row operation R1→ R1 — 3R2 in the following matrix equation:…
  78. ______ matrix is both symmetric and skew symmetric matrix. Fill in the blanks in…
  79. ______ matrix is both symmetric and skew symmetric matrix. Fill in the blanks in…
  80. Sum of two skew symmetric matrices is always _______ matrix. Fill in the blanks…
  81. Sum of two skew symmetric matrices is always _______ matrix. Fill in the blanks…
  82. The negative of a matrix is obtained by multiplying it by ________. Fill in the…
  83. The negative of a matrix is obtained by multiplying it by ________. Fill in the…
  84. The product of any matrix by the scalar _____ is the null matrix. Fill in the…
  85. A matrix which is not a square matrix is called a _____ matrix. Fill in the…
  86. A matrix which is not a square matrix is called a _____ matrix. Fill in the…
  87. Matrix multiplication is _____ over addition. Fill in the blanks in each of the…
  88. Matrix multiplication is _____ over addition. Fill in the blanks in each of the…
  89. If A is a symmetric matrix, then A^3 is a ______ matrix. Fill in the blanks in…
  90. If A is a symmetric matrix, then A^3 is a ______ matrix. Fill in the blanks in…
  91. If A is a skew symmetric matrix, then A^2 is a _________. Fill in the blanks in…
  92. If A is a skew symmetric matrix, then A^2 is a _________. Fill in the blanks in…
  93. If A and B are square matrices of the same order, then (i) (AB)’ = ________. (ii)…
  94. If A and B are square matrices of the same order, then (i) (AB)’ = ________. (ii)…
  95. If A is skew symmetric, then kA is a ______. (k is any scalar) Fill in the blanks…
  96. If A is skew symmetric, then kA is a ______. (k is any scalar) Fill in the blanks…
  97. If A and B are symmetric matrices, then (i) AB - BA is a _________. (ii) BA - 2AB…
  98. If A and B are symmetric matrices, then (i) AB - BA is a _________. (ii) BA - 2AB…
  99. If A is symmetric matrix, then B’AB is _______. Fill in the blanks in each of the…
  100. If A is symmetric matrix, then B’AB is _______. Fill in the blanks in each of the…
  101. If A and B are symmetric matrices of same order, then AB is symmetric if and only…
  102. If A and B are symmetric matrices of same order, then AB is symmetric if and only…
  103. In applying one or more now operations while finding A-1 by elementary row…
  104. In applying one or more now operations while finding A-1 by elementary row…
  105. A matrix denotes a number. Which of the following statements are True or False…
  106. A matrix denotes a number. Which of the following statements are True or False…
  107. Matrices of any order can be added. Which of the following statements are True or…
  108. Matrices of any order can be added. Which of the following statements are True or…
  109. Two matrices are equal if they have same number of rows and same number of…
  110. Matrices of different order cannot be subtracted. Which of the following…
  111. Matrices of different order cannot be subtracted. Which of the following…
  112. Matrix addition is associative as well as commutative. Which of the following…
  113. Matrix addition is associative as well as commutative. Which of the following…
  114. Matrix multiplication is commutative. Which of the following statements are True…
  115. Matrix multiplication is commutative. Which of the following statements are True…
  116. A square matrix where every element is unity is called an identity matrix. Which…
  117. A square matrix where every element is unity is called an identity matrix. Which…
  118. If A and B are two square matrices of the same order, then A + B = B + A. Which…
  119. If A and B are two square matrices of the same order, then A + B = B + A. Which…
  120. If A and B are two matrices of the same order, then A - B = B - A. Which of the…
  121. If A and B are two matrices of the same order, then A - B = B - A. Which of the…
  122. If matrix AB = O, then A = O or B = O or both A and B are null matrices. Which of…
  123. If matrix AB = O, then A = O or B = O or both A and B are null matrices. Which of…
  124. Transpose of a column matrix is a column matrix. Which of the following…
  125. Transpose of a column matrix is a column matrix. Which of the following…
  126. If A and B are two square matrices of the same order, then AB = BA. Which of the…
  127. If A and B are two square matrices of the same order, then AB = BA. Which of the…
  128. If each of the three matrices of the same order are symmetric, then their sum is…
  129. If A and B are any two matrices of the same order, then (AB)’ = A’B’. Which of…
  130. If A and B are any two matrices of the same order, then (AB)’ = A’B’. Which of…
  131. If (AB)’ = B’A’, where A and B are not square matrices, then number of rows in A…
  132. If (AB)’ = B’A’, where A and B are not square matrices, then number of rows in A…
  133. If A, B and C are square matrices of same order, then AB = AC always implies that…
  134. If A, B and C are square matrices of same order, then AB = AC always implies that…
  135. AA’ is always a symmetric matrix for any square matrix A. Which of the following…
  136. AA’ is always a symmetric matrix for any square matrix A. Which of the following…
  137. If a = [rrr 2&3&-1 1&4&2] b = [ll 2&3 4&5 2&1] then AB and BA are defined and…
  138. If a = [rrr 2&3&-1 1&4&2] b = [ll 2&3 4&5 2&1] then AB and BA are defined and…
  139. If A is skew symmetric matrix, then A^2 is a symmetric matrix. Which of the…
  140. If A is skew symmetric matrix, then A^2 is a symmetric matrix. Which of the…
  141. (AB)-1 = A-1.B-1, where A and B are invertible matrices satisfying cumulative…
  142. (AB)-1 = A-1.B-1, where A and B are invertible matrices satisfying cumulative…

Exercise
Question 1.

If a matrix has 28 elements, what are the possible orders it can have? What if it has 13 elements?


Answer:

We know that in mathematics, a matrix is a rectangular array of numbers, symbols, or expressions, arranged in rows and columns.

The number of rows and columns that a matrix has is called its order or its dimension. By convention, rows are listed first; and columns second.


We are given with a matrix that has 28 elements.


We know that,


If a matrix has mn elements, then the order of the matrix can be given by m × n, where m and n are natural numbers.


Therefore, for a matrix having 28 elements, that is, mn = 28, possible orders can be found out as follows:


∵ mn = 28


Take m and n to be any number, such that, when it is multiplied it gives 28.


So, let m = 1 and n = 28.


Then, m × n = 1 × 28 (=28)


⇒ 1 × 28 is a possible order of the matrix having 28 elements.


Take m = 2 and n = 14.


Then, m × n = 2 × 14 (=28)


⇒ 2 × 14 is a possible order of the matrix having 28 elements.


Take m = 4 and n = 7.


Then, m × n = 4 × 7 (=28)


⇒ 4 × 7 is a possible order of the matrix having 28 elements.


Take m = 7 and n = 4.


Then, m × n = 7 × 4 (=28)


⇒ 7 × 4 is a possible order of the matrix having 28 elements.


Take m = 14 and n = 2.


Then, m × n = 14 × 2 (=28)


⇒ 14 × 2 is a possible order of the matrix having 28 elements.


Take m = 28 and n = 1.


Then, m × n = 28 × 1 (=28)


⇒ 28 × 1 is a possible order of the matrix having 28 elements.


Thus, the possible orders of the matrix having 28 elements are


1 × 28, 2 × 14, 4 × 7, 7 × 4, 14 × 2 and 28 × 1


If the matrix had 13 elements, then also we find the possible order in same way.


Here, mn = 13.


Take m and n to be any number, such that, when it is multiplied it gives 13.


Take m = 1 and n = 13.


Then, m × n = 1 × 13 (=13)


⇒ 1 × 13 is a possible order of the matrix having 13 elements.


Take m = 13 and n = 1.


Then, m × n = 13 × 1 (=13)


⇒ 13 × 1 is a possible order of the matrix having 13 elements.


Thus, the possible orders of the matrix having 13 elements are


1 × 13 and 13 × 1



Question 2.

In the matrix , write:

(i) The order of the matrix A

(ii) The number of elements

(iii) Write elements a23, a31, a12


Answer:

We have the matrix


A matrix, as we know, is a rectangular array of numbers, symbols, or expressions, arranged in rows and columns.


(i). We need to find the order of the matrix A.


And we know that,


The number of rows and columns that a matrix has is called its order or its dimension. By convention, rows are listed first; and columns second.


So,


Here, in matrix A:


There are 3 rows.


Elements in 1st row = a, 1, x


Elements in 2nd row = 2, √3, x2 – y


Elements in 3rd row = 0, 5, -2/5


⇒ M = 3


And,


There are 3 columns.


Elements in 1st column = a, 2, 0


Elements in 2nd column = 1, √3, 5


Elements in 3rd column = x, x2 – y, -2/5


⇒ N = 3


Since, the order of matrix = M × N


⇒ The order of matrix A = 3 × 3


Thus, the order of the matrix A is 3 × 3.


(ii). We need to find the number of elements in the matrix A.


And we know that,


Each number that makes up a matrix is called an element of the matrix.


So,


If a matrix has M rows and N columns, the number of elements is MN.


Here, in matrix A:


There are 3 rows.


⇒ M = 3


And,


There are 3 columns.


⇒ N = 3


Then, number of elements = MN


⇒ Number of elements = 3 × 3


⇒ Number of elements = 9


The elements are namely, a, 2, 0, 1, √3, 5, x, x2 – y, -2/5.


Thus, the number of elements is 9.


(iii). We need to find the elements a23, a31 and a12.


We know that,


aij is the representation of elements lying in the ith row and jth column.


For a23:


Comparing aij with a23, we have


i = 2


j = 3


Look up in matrix A for element in (i=) 2nd row and (j=) 3rd column.



Element that is common to both 2nd row and 3rd column = x2 – y


⇒ a23 = x2 – y


For a31:


Comparing aij with a31, we have


i = 3


j = 1


Look up in matrix A for element in (i=) 3rd row and (j=) 1st column.



Element that is common to both 3rd row and 1st column = 0


⇒ a31 = 0


For a12:


Comparing aij with a12, we have


i = 1


j = 2


Look up in matrix A for element in (i=) 1st row and (j=) 2nd column.



Element that is common to both 1st row and 2nd column = 1


⇒ a12 = 1


Thus, a23 = x2 – y, a31 = 0 and a12 = 1.


Question 3.

Construct a2×2 matrix where

(i)

(ii) aij = |– 2i + 3j|


Answer:

We know that,

A matrix, as we know, is a rectangular array of numbers, symbols, or expressions, arranged in rows and columns.


Also,


We know that, the notation A = [aij]m×m indicates that A is a matrix of order m × n, also 1 ≤ i ≤ m, 1 ≤ j ≤ n; i, j ∈ N.


(i).We need to construct a matrix, a2×2, where



For a2×2,


1 ≤ i ≤ m


⇒ 1 ≤ i ≤ 2 [∵ m = 2]


And,


1 ≤ j ≤ n


⇒ 1 ≤ j ≤ 2 [∵ n = 2]


Put i = 1 and j = 1.






Put i = 1 and j = 2.






Put i = 2 and j = 1.





⇒ a21 = 0


Put i = 2 and j = 2.






⇒ a22 = 2


Let the matrix formed be A.



Substituting the values of a11, a12, a21 and a22, we get the matrix



(ii). We need to construct a matrix, a2×2, where


aij = |-2i + 3j|


For a2×2,


1 ≤ i ≤ m


⇒ 1 ≤ i ≤ 2 [∵ m = 2]


And,


1 ≤ j ≤ n


⇒ 1 ≤ j ≤ 2 [∵ n = 2]


Put i = 1 and j = 1.


a11 = |-2(1) + 3(1)|


⇒ a11 = |-2 + 3|


⇒ a11 = |1|


⇒ a11 = 1


Put i = 1 and j = 2.


a12 = |-2(1) + 3(2)|


⇒ a12 = |-2 + 6|


⇒ a12 = |4|


⇒ a12 = 4


Put i = 2 and j = 1.


a21 = |-2(2) + 3(1)|


⇒ a21 = |-4 + 3|


⇒ a21 = |-1|


⇒ a21 = 1


Put i = 2 and j = 2.


a22 = |-2(2) + 3(2)|


⇒ a22 = |-4 + 6|


⇒ a22 = |2|


⇒ a22 = 2


Let the matrix formed be A.



Substituting the values of a11, a12, a21 and a22, we get the matrix



Question 4.

Construct a 3 × 2 matrix whose elements are given by aij = eixsin jx


Answer:

A matrix, as we know, is a rectangular array of numbers, symbols, or expressions, arranged in rows and columns.

Also,


We know that, the notation A = [aij]m×m indicates that A is a matrix of order m × n, also 1 ≤ i ≤ m, 1 ≤ j ≤ n; i, j ∈ N.


We need to construct a 3 × 2 matrix whose elements are given by


aij = ei.x sin jx


For a3×2:


1 ≤ i ≤ m


⇒ 1 ≤ i ≤ 3 [∵ m = 3]


1 ≤ j ≤ n


⇒ 1 ≤ j ≤ 2 [∵ n = 2]


Put i = 1 and j = 1.


a11 = e(1)x sin (1)x


⇒ a11 = ex sin x


Put i = 1 and j = 2.


a12 = e(1)x sin (2)x


⇒ a12 = ex sin 2x


Put i = 2 and j = 1.


a21 = e(2)x sin (1)x


⇒ a21 = e2xsin x


Put i = 2 and j = 2.


a22 = e(2)x sin (2)x


⇒ a22 = e2x sin 2x


For i = 3 and j = 1.


a31 = e(3)x sin (1)x


⇒ a31 = e3x sin x


For i = 3 and j = 2.


a32 = e(3)x sin (2)x


⇒ a32 = e3x sin 2x


Let the matrix formed be A.



Substituting the values of a11, a12, a21, a22, a31 and a32, we get the matrix



Thus, we have got the matrix.


Question 5.

Find values of a and b if A = B, where

and


Answer:

We have the matrices A and B, where



We need to find the values of a and b.


We know that, if



Then,


a11 = b11


a12 = b12


a21 = b21


a22 = b22


Also, A = B.



This means,


a + 4 = 2a + 2 …(i)


3b = b2 + 2 …(ii)


8 = 8


-6 = b2 – 5b …(iii)


From equation (i), we can find the value of a.


a + 4 = 2a + 2


⇒ 2a – a = 4 – 2


⇒ a = 2


From equation (ii), we can find the value of b2.


3b = b2 + 2


⇒ b2= 3b – 2


Substitute the value of b2 in equation (iii), we get


-6 = b2 – 5b


⇒ -6 = (3b – 2) – 5b


⇒ -6 = 3b – 2 – 5b


⇒ -6 = 3b – 5b – 2


⇒ -6 = -2b – 2


⇒ 2b = 6 – 2


⇒ 2b = 4



⇒ b = 2


Thus, a = 2 and b = 2.



Question 6.

If possible, find the sum of the matrices A and B, where and .


Answer:

We know that,

The number of rows and columns that a matrix has is called its order or its dimension. By convention, rows are listed first; and columns second.


Also,


Addition or subtraction of matrices is possible only if the matrices are of same order.


That is,


If A and B are two matrices and if they are needed to be added, then if order of A is m × n, order of B must be m × n.


We have matrices A and B, where




We know what order of matrix is,


If a matrix has M rows and N columns, the order of matrix is M × N.


In matrix A:


Number of rows = 2


⇒ M = 2


Number of column = 2


⇒ N = 2


Then, order of matrix A = M × N


⇒ Order of matrix A = 2 × 2


In matrix B:


Number of rows = 2


⇒ M = 2


Number of columns = 3


⇒ M = 3


Then, order of matrix B = M × N


⇒ order of matrix B = 2 × 3


Since,


Order of matrix A ≠ Order of matrix B


⇒ Matrices A and B cannot be added.


Thus, matrix A and matrix B cannot be added.



Question 7.

If and find

(i) X + Y

(ii) 2X – 3Y

(iii) A matrix Z such that X + Y + Z is a zero matrix.


Answer:

Addition or subtraction of matrices is possible only if the matrices are of same order.

That is,


If A and B are two matrices and if they are needed to be added, then if order of A is m × n, order of B must be m × n.


We have matrices X and Y, where




We know what order of matrix is,


If a matrix has M rows and N columns, the order of matrix is M × N.


(i). We need to find the X + Y.


Let us first determine order of X and Y.


Order of X:


Number of rows = 2


⇒ M = 2


Number of columns = 3


⇒ N = 3


Then, order of matrix X = M × N


⇒ Order of matrix X = 2 × 3


Order of Y:


Number of rows = 2


⇒ M = 2


Number of columns = 3


⇒ N = 3


Then, order of matrix Y = M × N


⇒ Order of matrix Y = 2 × 3


Since, order of matrix X = order of matrix Y


⇒ Matrices X and Y can be added.


So,





Thus, .


(ii). We need to find 2X – 3Y.


Let us calculate 2X.


We have,



Then, multiplying by 2 on both sides, we get





Also,



Multiplying by 3 on both sides, we get





Now subtract 3Y from 2X.





Thus, .


(iii). We need to find matrix Z, such that X + Y + Z is a zero matrix.


That is,


X + Y + Z = 0


Or,


Z = -X – Y


Or,


Z = -(X + Y)


We have already found X + Y in part (i).


So, from part (i):



Then,




Thus, .



Question 8.

Find non-zero values of x satisfying the matrix equation:



Answer:

A matrix, as we know, is a rectangular array of numbers, symbols, or expressions, arranged in rows and columns.

Also,


Two or more matrices can be added or subtracted only if they have same order.


And we are familiar with order of a matrix. If a matrix has M rows and N columns, the order of matrix is M × N.


We have matrix equation,



Take matrix .


Multiply it with x,



…(i)


Take matrix .


Multiply it with 2,



…(ii)


Take matrix .


Multiply it with 2,



…(iii)


Add equation (i) and (ii) and make it equal to equation (iii), we get




Adding left side of the matrix equation as they have same order.



We need to find the value of x.


So, compare the elements in the two matrices.


If,



Then,


a11 = b11


a12 = b12


a21 = b21


a22 = b22


So,


2x2 + 16 = 2(x2 + 8) …(i)


2x + 10x = 48 …(ii)


3x + 8 = 20 …(iii)


x2 + 8x = 12x …(iv)


We have got equations (i), (ii), (iii) and (iv) to solve for x.


So, take equation (i).


2x2 + 16 = 2x2 + 16


We won’t be able to find x from this equation, as both equations are same.


Now, take equation (ii).


2x + 10x = 48


⇒ 12x = 48



⇒ x = 4


From equation (iii),


3x + 8 = 20


⇒ 3x = 20 – 8


⇒ 3x = 12



⇒ x = 4


From equation (iv),


x2 + 8x = 12x


⇒ x2 = 12x – 8x


⇒ x2 = 4x


⇒ x2 – 4x = 0


⇒ x(x – 4) = 0


⇒ x = 0 or (x – 4) = 0


⇒ x = 0 or x = 4


⇒ x = 4 (∵ x = 0 does not satisfy equations (ii) and (iii))


So, by solving equations (ii), (iii) and (iv), we can conclude that


x = 4


Thus, the value of x is 4.



Question 9.

If and , show that (A + B) (A – B) ≠ A2 – B2.


Answer:

We have the matrices A and B, where



We need to show that (A + B) (A – B) ≠ A2 – B2.


Take L.H.S: (A + B) (A – B)


First, let us compute (A + B).


If two matrices are of same order (say, m × n), then they can be added or subtracted. Example,


If we have matrices and . Then, they can be added as



So,





Now, let us compute (A – B).


Similarly, two matrices having same order can be subtracted in a similar fashion.


So,






Now, let us compute (A + B) (A – B).


In order to multiply two matrices, A and B, the number of columns in A must equal the number of rows in B. Thus, if A is an m x n matrix and B is an r x s matrix, n = r.



Multiply 1st row of matrix A by matching members of 1st column of matrix B, then sum them up.


(0, 0).(0, 0) = (0 × 0) + (0 × 0)


⇒ (0, 0).(0, 0) = 0 + 0


⇒ (0, 0).(0, 0) = 0



Multiply 1st row of matrix A by matching members of 2nd column of matrix B, then sum them up.


(0, 0).(2, 1) = (0 × 2) + (0 × 1)


⇒ (0, 0).(2, 1) = 0 + 0


⇒ (0, 0).(2, 1) = 0



Multiply 2nd row of matrix A by matching members of 1st column of matrix B, then sum them up.


(2, 1).(0, 0) = (2 × 0) + (1 × 0)


⇒ (2, 1).(0, 0) = 0 + 0


⇒ (2, 1).(0, 0) = 0



Multiply 2nd row of matrix A by matching members of 2nd column of matrix B, then sum them up.


(2, 1).(2, 1) = (2 × 2) + (1 × 1)


⇒ (2, 1).(2, 1) = 4 + 1


⇒ (2, 1).(2, 1) = 5



So, we have



Take R.H.S: A2 – B2


Let us compute A2 first.


A2 = A.A


So, we need to compute A.A.



Multiply 1st row of matrix A by matching members of 1st column of matrix A, then sum them up.


(0, 1).(0, 1) = (0 × 0) + (1 × 1)


⇒ (0, 1).(0, 1) = 0 + 1


⇒ (0, 1).(0, 1) = 1



Multiply 1st row of matrix A by matching members of 2nd column of matrix A, then sum them up.


(0, 1).(1, 1) = (0 × 1) + (1 × 1)


⇒ (0, 1).(1, 1) = 0 + 1


⇒ (0, 1).(1, 1) = 1



Multiply 2nd row of matrix A by matching members of 1st column of matrix A, then sum them up.


(1, 1).(0, 1) = (1 × 0) + (1 × 1)


⇒ (1, 1).(0, 1) = 0 + 1


⇒ (1, 1).(0, 1) = 1



Multiply 2nd row of matrix A by matching members of 2nd column of matrix A, then sum them up.


(1, 1).(1, 1) = (1 × 1) + (1 × 1)


⇒ (1, 1).(1, 1) = 1 + 1


⇒ (1, 1).(1, 1) = 2



So,



Now, let us compute B2.


B2 = B.B


We need to compute B.B.



Multiply 1st row of matrix B by matching members of 1st column of matrix B, then sum them up.


(0, -1).(0, 1) = (0 × 0) + (-1 × 1)


⇒ (0, -1).(0, 1) = 0 – 1


⇒ (0, -1).(0, 1) = -1



Multiply 1st row of matrix B by matching members of 2nd column of matrix B, then sum them up.


(0, -1).(-1, 0) = (0 × -1) + (-1 × 0)


⇒ (0, -1).(-1, 0) = 0 + 0


⇒ (0, -1).(-1, 0) = 0



Multiply 2nd row of matrix B by matching members of 1st column of matrix B, then sum them up.


(1, 0).(0, 1) = (1 × 0) + (0 × 1)


⇒ (1, 0).(0, 1) = 0 + 0


⇒ (1, 0).(0, 1) = 0



Multiply 2nd row of matrix B by matching members of 2nd column of matrix B, then sum them up.


(1, 0).(-1, 0) = (1 × -1) + (0 × 0)


⇒ (1, 0).(-1, 0) = -1 + 0


⇒ (1, 0).(-1, 0) = -1



So,



Now, compute A2 – B2.






Clearly,


and are not equal.


Thus, (A + B)(A – B) ≠ A2 – B2.



Question 10.

Find the value of x if



Answer:

We have the matrix equation,


We need to find the value of x.


Let us compute L.H.S:


Let,


and



In order to multiply two matrices, A and B, the number of columns in A must equal the number of rows in B. Thus, if A is an m x n matrix and B is an r x s matrix, n = r.


First, let us compute



Multiply 1st row of matrix A by matching members of 1st column of matrix B, then sum them up.


(1, x, 1).(1, 2, 15) = (1 × 1) + (x × 2) + (1 × 15)


⇒ (1, x, 1).(1, 2, 15) = 1 + 2x + 15


⇒ (1, x, 1).(1, 2, 15) = 2x + 16



Multiply 1st row of matrix A by matching members of 2nd column of matrix B, then sum them up.


(1, x, 1).(3, 5, 3) = (1 × 3) + (x × 5) + (1 × 3)


⇒ (1, x, 1).(3, 5, 3) = 3 + 5x + 3


⇒ (1, x, 1).(3, 5, 3) = 5x + 6



Multiply 1st row of matrix A by matching members of 3rd column of matrix B, then sum them up.


(1, x, 1).(2, 1, 2) = (1 × 2) + (x × 1) + (1 × 2)


⇒ (1, x, 1).(2, 1, 2) = 2 + x + 2


⇒ (1, x, 1).(2, 1, 2) = x + 4



So,



Now, compute



Multiply 1st row of matrix D by matching members of 1st column of matrix C, then sum them up.


(2x + 16, 5x + 6, x + 4).(1, 2, x) = ((2x + 16) × 1) + ((5x + 6) × 2) + ((x + 4) × x)


⇒ (2x + 16, 5x + 6, x + 4).(1, 2, x) = (2x + 16) + (10x + 12) + (x2 + 4x)


⇒ (2x + 16, 5x + 6, x + 4).(1, 2, x) = x2 + 2x + 10x + 4x + 16 + 12


⇒ (2x + 16, 5x + 6, x + 4).(1, 2, x) = x2 + 16x + 28



So, we have got



Now, put L.H.S = R.H.S


[x2 + 16x + 28] = [0]


This means,


x2 + 16x + 28 = 0


⇒ x2 + 14x + 2x + 28 = 0


⇒ x(x + 14) + 2(x + 14) = 0


⇒ (x + 2)(x + 14) = 0


⇒ (x + 2) = 0 or (x + 14) = 0


⇒ x = -2 or x = -14


Thus, x = -2, -14.



Question 11.

Show that satisfies the equation A2 – 3A – 7I = 0 and hence find A–1.


Answer:

We have the matrix A, such that


(i). We need to show that the matrix A satisfies the equation A2 – 3A – 7I = 0.


(ii). Also, we need to find A-1.


(i). Take L.H.S: A2 – 3A – 7I


First, compute A2.


A2 = A.A



In order to multiply two matrices, A and B, the number of columns in A must equal the number of rows in B. Thus, if A is an m x n matrix and B is an r x s matrix, n = r.


Multiply 1st row of matrix A by matching members of 1st column of matrix A, then sum them up.


(5, 3).(5, -1) = (5 × 5) + (3 × -1)


⇒ (5, 3).(5, -1) = 25 + (-3)


⇒ (5, 3).(5, -1) = 25 – 3


⇒ (5, 3).(5, -1) = 22



Multiply 1st row of matrix A by matching members of 2nd column of matrix A, then sum them up.


(5, 3).(3, -2) = (5 × 3) + (3 × -2)


⇒ (5, 3).(3, -2) = 15 + (-6)


⇒ (5, 3).(3, -2) = 15 – 6


⇒ (5, 3).(3, -2) = 9



Multiply 2nd row of matrix A by matching members of 1st column of matrix A, then sum them up.


(-1, -2).(5, -1) = (-1 × 5) + (-2 × -1)


⇒ (-1, -2).(5, -1) = -5 + 2


⇒ (-1, -2).(5, -1) = -3



Multiply 2nd row of matrix A by matching members of 2nd column of matrix A, then sum them up.


(-1, -2).(3, -2) = (-1 × 3) + (-2 × -2)


⇒ (-1, -2).(3, -2) = -3 + 4


⇒ (-1, -2).(3, -2) = 1




Substitute values of A2 and A in A2 – 3A – 7I.



Also, since matrix A is of the order 2 × 2, then I will be the identity matrix of order 2 × 2 such that,








Clearly,


L.H.S = R.H.S


Thus, we have shown that matrix A satisfy A2 – 3A – 7I = 0.


(ii). Now, let us find A-1.


We know that, inverse of matrix A is A-1 is true only when


A × A-1 = A-1 × A = I


Where, I = Identity matrix


We have,


A2 – 3A – 7I = 0


Multiply A-1 on both sides, we get


A-1(A2 – 3A – 7I) = A-1 × 0


⇒ A-1.A2 – A-1.3A – A-1.7I = 0


⇒ A-1.A.A – 3A-1.A – 7A-1.I = 0


⇒ (A-1A)A – 3(A-1A) – 7(A-1I) = 0


And as A-1A = I and A-1I = A-1


⇒ IA – 3I – 7A-1 = 0


Since, IA = A


⇒ A – 3I – 7A-1 = 0


⇒ 7A-1 = A – 3I



[∵ ]





Thus, .



Question 12.

Find the matrix A satisfying the matrix equation:



Answer:

Here we have been given a matrix equation,


We need to find the matrix A.


Let matrix A be of order 2 × 2, and can be represented as



Then, we get



Take L.H.S:


So, first let us calculate


In order to multiply two matrices, A and B, the number of columns in A must equal the number of rows in B. Thus, if A is an m x n matrix and B is an r x s matrix, n = r.


Multiply 1st row of matrix X by matching members of 1st column of matrix Y, then sum them up.


(2, 1).(a, c) = (2 × a) + (1 × c)


⇒ (2, 1).(a, c) = 2a + c



Multiply 1st row of matrix X by matching members of 2nd column of matrix Y, then sum them up.


(2, 1).(b, d) = (2 × b) + (1 × d)


⇒ (2, 1).(b, d) = 2b + d



Multiply 2nd row of matrix X by matching members of 1st column of matrix Y, then sum them up.


(3, 2).(a, c) = (3 × a) + (2 × c)


⇒ (3, 2).(a, c) = 3a + 2c



Multiply 2nd row of matrix X by matching members of 2nd column of matrix Y, then sum them up.


(3, 2).(b, d) = (3 × b) + (2 × d)


⇒ (3, 2).(b, d) = 3b + 2d



Let X.Y = Z


Now, we need to find .


That is,



Where, let .


Multiply 1st row of matrix Z by matching members of 1st column of matrix Q, then sum them up.


(2a + c, 2b + d).(-3, 5) = ((2a + c) × -3) + ((2b + d) × 5)


⇒ (2a + c, 2b + d).(-3, 5) = -6a – 3c + 10b + 5d


⇒ (2a + c, 2b + d).(-3, 5) = -6a + 10b – 3c + 5d



Multiply 1st row of matrix Z by matching members of 2nd column of matrix Q, then sum them up.


(2a + c, 2b + d).(2, -3) = ((2a + c) × 2) + ((2b + d) × -3)


⇒ (2a + c, 2b + d).(2, -3) = 4a + 2c – 6b – 3d


⇒ (2a + c, 2b + d).(2, -3) = 4a – 6b + 2c – 3d



Multiply 2nd row of matrix Z by matching members of 1st column of matrix Q, then sum them up.


(3a + 2c, 3b + 2d).(-3, 5) = ((3a + 2c) × -3) + ((3b + 2d) × 5)


⇒ (3a + 2c, 3b + 2d).(-3, 5) = -9a – 6c + 15b + 10d


⇒ (3a + 2c, 3b + 2d).(-3, 5) = -9a + 15b – 6c + 10d



Multiply 2nd row of matrix Z by matching members of 2nd column of matrix Q, then sum them up.


(3a + 2c, 3b + 2d).(2, -3) = ((3a + 2c) × 2) + ((3b + 2d) × -3)


⇒ (3a + 2c, 3b + 2d).(2, -3) = 6a + 4c – 9b – 6d


⇒ (3a + 2c, 3b + 2d).(2, -3) = 6a – 9b + 4c – 6d



So, we have



Now, for L.H.S = R.H.S



For matrices having same order, we can write as


-6a + 10b – 3c + 5d = 1 …(i)


4a – 6b + 2c – 3d = 0 …(ii)


-9a + 15b – 6c + 10d = 0 …(iii)


6a – 9b + 4c – 6d = 1 …(iv)


We have 4 variables to find, namely, a, b, c and d; and 4 equations.


So, on adding equations (i) and (iv), we get


(-6a + 10b – 3c + 5d) + (6a – 9b + 4c – 6d) = 1 + 1


⇒ -6a + 6a + 10b – 9b – 3c + 4c + 5d – 6d = 2


⇒ 0 + b + c – d = 2


⇒ d = b + c – 2 …(v)


Now, adding equations (ii) and (iii), we get


(4a – 6b + 2c – 3d) + (-9a + 15b – 6c + 10d) = 0 + 0


⇒ 4a – 9a – 6b + 15b + 2c – 6c – 3d + 10d = 0


⇒ -5a + 9b – 4c + 7d = 0 …(vi)


On adding equations (iv) and (vi), we get


(6a – 9b + 4c – 6d) + (-5a + 9b – 4c + 7d) = 1 + 0


⇒ 6a – 5a – 9b + 9b + 4c – 4c – 6d + 7d = 1


⇒ a + 0 + 0 + d = 1


⇒ d = 1 – a …(vii)


Putting the value of d from equation (vii) in (v), we get


(1 – a) = b + c – 2


⇒ b + c – 2 – 1 = -a


⇒ b + c – 3 = -a


⇒ a = 3 – b – c …(viii)


Now, putting values of a and d from equations (vii) and (viii) in equation (iii), we get


-9(3 – b – c) + 15b – 6c + 10(1 – a) = 0


⇒ -9(3 – b – c) + 15b – 6c + 10(1 – (3 – b – c)) = 0 [∵ a = 3 – b – c]


⇒ -27 + 9b + 9c + 15b – 6c + 10(1 – 3 + b + c) = 0


⇒ -27 + 9b + 9c + 15b – 6c + 10(-2 + b + c) = 0


⇒ -27 + 9b + 9c + 15b – 6c – 20 + 10b + 10c = 0


⇒ 9b + 15b + 10b + 9c – 6c + 10c – 27 – 20 = 0


⇒ 34b + 13c – 47 = 0


⇒ 34b + 13c = 47 …(ix)


Also, putting values of a and d from equations (vii) and (viii) in equation (ii), we get


4(3 – b – c) – 6b + 2c – 3(1 – a) = 0


⇒ 12 – 4b – 4c – 6b + 2c – 3(1 – (3 – b – c)) = 0


⇒ 12 – 4b – 4c – 6b + 2c – 3(1 – 3 + b + c) = 0


⇒ 12 – 4b – 4c – 6b + 2c – 3(-2 + b + c) = 0


⇒ 12 – 4b – 4c – 6b + 2c + 6 – 3b – 3c = 0


⇒ -4b – 6b – 3b – 4c + 2c – 3c + 12 + 6 = 0


⇒ -13b – 5c + 18 = 0


⇒ 13b + 5c = 18 …(x)


On multiplying equation (ix) by 5 and equation (x) by 13, we get


(ix) ⇒ 5(34b + 13c) = 5 × 47


⇒ 170b + 65c = 235 …(xi)


(x) ⇒ 13(13b + 5c) = 13 × 18


⇒ 169b + 65c = 234 …(xii)


Subtracting equations (xi) and (xii), we get


(170b + 65c) – (169b + 65c) = 235 – 234


⇒ 170b – 169b + 65c – 65c = 1


⇒ b = 1


Putting b = 1 in equation (x), we get


13(1) + 5c = 18


⇒ 13 + 5c = 18


⇒ 5c = 18 – 13


⇒ 5c = 5



⇒ c = 1


Putting b = 1 and c = 1 in equation (viii), we get


a = 3 – b – c


⇒ a = 3 – 1 – 1


⇒ a = 3 – 2


⇒ a = 1


Putting a = 1 in equation (vii), we get


d = 1 – a


⇒ d = 1 – 1


⇒ d = 0


Thus, the matrix A is




Question 13.

Find A, if .


Answer:

We have,


We need to find the matrix A.


Let us see what the order of the matrices given are.


We know what order of matrix is,


If a matrix has M rows and N columns, the order of matrix is M × N.


Order of .


Number of rows = 3


⇒ M = 3


Number of column = 1


⇒ N = 1


Then, order of matrix X = M × N


⇒ Order of matrix X = 3 × 1


Order of .


Number of rows = 3


⇒ M = 3


Number of columns = 3


⇒ N = 3


Then, order of matrix Y = M × N


⇒ Order of matrix Y = 3 × 3


We must understand that, when a matrix of order 1 × 3 is multiplied to the matrix X, only then matrix Y is produced.


Let matrix A be of order 1 × 3, and can be represented as



Then, we get



Take L.H.S:


In order to multiply two matrices, A and B, the number of columns in A must equal the number of rows in B. Thus, if A is an m x n matrix and B is an r x s matrix, n = r.


So, we have



Multiply 1st row of matrix X by matching member of 1st column of matrix A, then sum them up.


(4)(a) = 4a



Multiply 1st row of matrix X by matching member of 2nd column of matrix A, then sum them up.


(4)(b) = 4b



Multiply 1st row of matrix X by matching member of 3rd column of matrix A, then sum them up.


(4)(c) = 4c



Multiply 2nd row of matrix X by matching member of 1st column of matrix A, then sum them up.


(1)(a) = a



Multiply 2nd row of matrix X by matching member of 2nd column of matrix A, then sum them up.


(1)(b) = b



Multiply 2nd row of matrix X by matching member of 3rd column of matrix A, then sum them up.


(1)(c) = c



Multiply 3rd row of matrix X by matching member of 1st column of matrix A, then sum them up.


(3)(a) = 3a



Multiply 3rd row of matrix X by matching member of 2nd column of matrix A, then sum them up.


(3)(b) = 3b



Multiply 3rd row of matrix X by matching member of 3rd column of matrix A, then sum them up.


(3)(c) = 3c



Now, L.H.S = R.H.S




Since, the matrices on either sides are of same order, we can say that


4a = -4 …(i)


4b = 8 …(ii)


4c = 4 …(iii)


a = -1 …(iv)


b = 2 …(v)


c = 1 …(vi)


3a = -3 …(vii)


3b = 6 …(viii)


3c = 3 …(ix)


From equation (i), we can find the value of a,


4a = -4



⇒ a = -1


From equation (ii), we can find the value of b,


4b = 8



⇒ b = 2


From equation (iii), we can find the value of c,


4c = 4



⇒ c = 1


And it will satisfy other equations (iv), (v), (vi), (vii), (viii) and (ix) too.


Thus, the matrix A is




Question 14.

If and , then verify (BA)2 ≠ B2A2.


Answer:

We have,



We need to verify (BA)2 ≠ B2A2.


Take L.H.S: (BA)2


First, compute BA.



We know what order of matrix is,


If a matrix has M rows and N columns, the order of matrix is M × N.


Order of matrix B:


Number of rows = 2


⇒ M = 2


Number of columns = 3


⇒ N = 3


Then, order of matrix = M × N


⇒ Order of matrix B = 2 × 3


Order of matrix A:


Number of rows = 3


⇒ M = 3


Number of columns = 2


⇒ N = 2


Then, order of matrix = M × N


⇒ Order of matrix A = 3 × 2


Since, in order to multiply two matrices, A and B, the number of columns in A must equal the number of rows in B. Thus, if A is an m x n matrix and B is an r x s matrix, n = r.


So, A and B can be multiplied.



Multiply 1st row of matrix B by matching member of 1st column of matrix A, then sum them up.


(2, 1, 2)(3, 1, 2) = (2 × 3) + (1 × 1) + (2 × 2)


⇒ (2, 1, 2)(3, 1, 2) = 6 + 1 + 4


⇒ (2, 1, 2)(3, 1, 2) = 11



Multiply 1st row of matrix B by matching member of 2nd column of matrix A, then sum them up.


(2, 1, 2)(-4, 1, 0) = (2 × -4) + (1 × 1) + (2 × 0)


⇒ (2, 1, 2)(-4, 1, 0) = -8 + 1 + 0


⇒ (2, 1, 2)(-4, 1, 0) = -7



Multiply 2nd row of matrix B by matching member of 1st column of matrix A, then sum them up.


(1, 2, 4)(3, 1, 2) = (1 × 3) + (2 × 1) + (4 × 2)


⇒ (1, 2, 4)(3, 1, 2) = 3 + 2 + 8


⇒ (1, 2, 4)(3, 1, 2) = 13



Multiply 2nd row of matrix B by matching member of 2nd column of matrix A, then sum them up.


(1, 2, 4)(-4, 1, 0) = (1 × -4) + (2 × 1) + (4 × 0)


⇒ (1, 2, 4)(-4, 1, 0) = -4 + 2 + 0


⇒ (1, 2, 4)(-4, 1, 0) = -2



So,


(BA)2 = (BA).(BA)



Similarly,





Take R.H.S: B2A2


Let us first compute B2.


B2 = B.B



In order to multiply two matrices, A and B, the number of columns in A must equal the number of rows in B. Thus, if A is an m x n matrix and B is an r x s matrix, n = r.


Note that in matrix B, number of columns ≠ number of rows.


This means, we can’t find B2.


⇒ L.H.S ≠ R.H.S


Thus, (BA)2 ≠ B2A2.



Question 15.

If possible, find BA and AB, where



Answer:

We are given matrices A and B, such that



We are required to find BA and AB, if possible.


Since, in order to multiply two matrices, A and B, the number of columns in A must equal the number of rows in B. Thus, if A is an m x n matrix and B is an r x s matrix, n = r.


Let us check for BA.



If a matrix has M rows and N columns, the order of matrix is M × N.


Order of B:


Number of rows = 3


⇒ M = 3


Number of columns = 2


⇒ N = 2


Then, order of matrix B = M × N


⇒ Order of matrix B = 3 × 2


Order of A:


Number of rows = 2


⇒ M = 2


Number of columns = 3


⇒ N = 3


Then, order of matrix A = M × N


⇒ Order of matrix A = 2 × 3


Here,


Number of columns in matrix B = Number of rows in matrix A = 2


So, BA is possible.


Let us check for AB.



Here,


Number of columns in matrix A = Number of rows in matrix B = 3


So, AB is also possible.


Let us find out BA.



Multiply 1st row of matrix B by matching members of 1st column of matrix A, then sum them up.


(4, 1).(2, 1) = (4 × 2) + (1 × 1)


⇒ (4, 1).(2, 1) = 8 + 1


⇒ (4, 1).(2, 1) = 9



Multiply 1st row of matrix B by matching members of 2nd column of matrix A, then sum them up.


(4, 1).(1, 2) = (4 × 1) + (1 × 2)


⇒ (4, 1).(1, 2) = 4 + 2


⇒ (4, 1).(1, 2) = 6



Similarly, let us calculate in the matrix itself.





Now, let us find out AB.



Multiply 1st row of matrix A by matching members of 1st column of matrix B, then sum them up.


(2, 1, 2).(4, 2, 1) = (2 × 4) + (1 × 2) + (2 × 1)


⇒ (2, 1, 2).(4, 2, 1) = 8 + 2 + 2


⇒ (2, 1, 2).(4, 2, 1) = 12



Multiply 1st row of matrix A by matching members of 2nd column of matrix B, then sum them up.


(2, 1, 2).(1, 3, 2) = (2 × 1) + (1 × 3) + (2 × 2)


⇒ (2, 1, 2).(1, 3, 2) = 2 + 3 + 4


⇒ (2, 1, 2).(1, 3, 2) = 9



Similarly, let us calculate in the matrix itself.





Thus, and .



Question 16.

Show by an example that for A ≠ O, B ≠ O, AB = O.


Answer:

We know that,

In order to multiply two matrices, A and B, the number of columns in A must equal the number of rows in B. Thus, if A is an m x n matrix and B is an r x s matrix, n = r.


We are given that,


A ≠ 0 and B ≠ 0


We need to show that, AB = 0.


For multiplication of A and B,


Number of columns of matrix A = Number of rows of matrix B = 2 (let)


Matrices A and B are square matrices of order 2 × 2.


For AB to become 0, one of the column of matrix A and other row of matrix B must be 0.


For example,




Check: Multiply AB.



Multiply 1st row of matrix A by matching members of 1st column of matrix B, then sum them up.


(0, 1).(3, 0) = (0 × 3) + (1 × 0)


⇒ (0, 1).(3, 0) = 0 + 0 = 0



Similarly, let us do it for the rest of the elements.




Thus, this example justifies the criteria.



Question 17.

Given and . Is (AB)’ = B’A’?


Answer:

We have two matrices A and B, such that



We need to verify whether (AB)’ = B’A’.


Let us understand what a transpose is.


In linear algebra, the transpose of a matrix is an operator which flips a matrix over its diagonal, that is it switches the row and column indices of the matrix by producing another matrix denoted as AT.


Take L.H.S = (AB)’


So, let us compute AB.



Multiply 1st row of matrix A by matching members of 1st column of matrix B, then sum them up.


(2, 4, 0)(1, 2, 1) = (2 × 1) + (4 × 2) + (0 × 1)


⇒ (2, 4, 0)(1, 2, 1) = 2 + 8 + 0


⇒ (2, 4, 0)(1, 2, 1) = 10



Multiply 1st row of matrix A by matching members of 2nd column of matrix B, then sum them up.


(2, 4, 0)(4, 8, 3) = (2 × 4) + (4 × 8) + (0 × 3)


⇒ (2, 4, 0)(4, 8, 3) = 8 + 32 + 0


⇒ (2, 4, 0)(4, 8, 3) = 40



Similarly, let us do it for the rest of the elements.





So,



Now, for transpose of AB, rows will become columns.



Now, take R.H.S = B’A’


If


Then, if (1, 4) are the elements of 1st row, it will become elements of 1st column, and so on.



Also,



Then, if (2, 4, 0) are the elements of 1st row, it will become elements of 1st column, and so on.



Now, multiply B’A’.



Multiply 1st row of matrix B’ by matching members of 1st column of matrix A’, then sum them up.


(1, 2, 1)(2, 4, 0) = (1 × 2) + (2 × 4) + (1 × 0)


⇒ (1, 2, 1)(2, 4, 0) = 2 + 8 + 0


⇒ (1, 2, 1)(2, 4, 0) = 10



Multiply 1st row of matrix B’ by matching members of 2nd column of matrix A’, then sum them up.


(1, 2, 1)(3, 9, 6) = (1 × 3) + (2 × 9) + (1 × 6)


⇒ (1, 2, 1)(3, 9, 6) = 3 + 18 + 6


⇒ (1, 2, 1)(3, 9, 6) = 27



Similarly, filling up for the rest of the elements.





⇒ L.H.S = R.H.S


Thus, (AB)’ = B’A’.



Question 18.

Solve for x and y:



Answer:

We are given with a matrix equation,


We need to find x and y.




These matrices can be added easily as they are of same order.



If two matrices are equal, then their corresponding elements are also equal.


This implies,


2x + 3y – 8 = 0 …(i)


x + 5y – 11 = 0 …(ii)


We have two variables, x and y; and two equations. It can be solved.


Rearranging equation (i), we get


2x + 3y = 8 …(iii)


Rearranging equation (ii), then multiplying it by 2 on both sides, we get


x + 5y = 11


2(x + 5y) = 2 × 11


⇒ 2x + 10y = 22 …(iv)


Subtracting equation (iii) from (iv), we get


(2x + 10y) – (2x + 3y) = 22 – 8


⇒ 2x + 10y – 2x – 3y = 14


⇒ 2x – 2x + 10y – 3y = 14


⇒ 7y = 14



⇒ y = 2


Substituting y = 2 in equation (iii), we get


2x + 3(2) = 8


⇒ 2x + 6 = 8


⇒ 2x = 8 – 6


⇒ 2x = 2



⇒ x = 1


Thus, x = 1 and y = 2.



Question 19.

If X and Y are 2 × 2 matrices, then solve the following matrix equations for X and Y

,


Answer:

We have the matrix equations,

…(i)


…(ii)


Subtracting equation (i) from (ii), we get





…(iii)


Adding equations (i) and (ii), we get









…(iv)


Adding equations (iii) and (iv), we get









Putting the matrix A in equation (iv), we get







Thus, and .



Question 20.

If A = [3 5], B = [7 3], then find a non-zero matrix C such that AC = BC.


Answer:

We have the matrices A and B, such that



We need to find matric C, such that AC = BC.


Let C be a non-zero matrix of order 2 × 1, such that



But order of C can be 2 × 1, 2 × 2, 2 × 3, 2 × 4, …


[∵ In order to multiply two matrices, A and B, the number of columns in A must equal the number of rows in B. Thus, if A is an m x n matrix and B is an r x s matrix, n = r.


∴, number of columns in matrix A = number of rows in matrix C = 2]


Take AC.



Multiply 1st row of matrix A by matching members of 1st column of matrix C, then sum them up.


(3, 5)(x, y) = (3 × x) + (5 × y)


⇒ (3, 5)(x, y) = 3x + 5y




Now, take BC.



Multiply 1st row of matrix B by matching members of 1st column of matrix C, then sum them up.


(7, 3)(x, y) = (7 × x) + (3 × y)


⇒ (7, 3)(x, y) = 7x + 3y




And,


AC = BC


⇒ [3x + 5y] = [7x + 3y]


⇒ 3x + 5y = 7x + 3y


⇒ 7x – 3x = 5y – 3y


⇒ 4x = 2y


⇒ y = 2x


Then,



Since, C is of orders, 2 × 1, 2 × 2, 2 × 3, …



In general,



Where, k is any real number.



Question 21.

Given an example of matrices A, B and C such that AB = AC, where A is non-zero matrix, but B ≠ C.


Answer:

We need to form matrices A, B and C such that AB = AC, where A is a non-zero matrix, but B ≠ C.

Take,





First, compute AB.



Multiply 1st row of matrix A by matching members of 1st column of matrix B, then sum them up.


(1, 0)(1, 2) = (1 × 1) + (0 × 2)


⇒ (1, 0)(1, 2) = 1 + 0


⇒ (1, 0)(1, 2) = 1



Similarly, let us do the same for other elements.





Now, let us compute AC.



Multiply 1st row of matrix A by matching members of 1st column of matrix C, then sum them up.


(1, 0)(1, 2) = (1 × 1) + (0 × 2)


⇒ (1, 0)(1, 2) = 1 + 0


⇒ (1, 0)(1, 2) = 1



Similarly, let us do the same for other elements.





Clearly, AB = AC.


Thus, we have found an example that satisfy the required criteria.



Question 22.

If , and , verify:

(i) (AB) C = A (BC)

(ii) A(B + C) = AB + AC


Answer:

We have matrices A, B and C, such that




In order to multiply two matrices, A and B, the number of columns in A must equal the number of rows in B. Thus, if A is an m x n matrix and B is an r x s matrix, n = r.


(i). We need to verify: (AB)C = A(BC)


Take L.H.S = (AB)C


First, compute AB.



Multiply 1st row of matrix A by matching members of 1st column of matrix B, then sum them up.


(1, 2)(2, 3) = (1 × 2) + (2 × 3)


⇒ (1, 2)(2, 3) = 2 + 6


⇒ (1, 2)(2, 3) = 8



Multiply 1st row of matrix A by matching members of 2nd column of matrix B, then sum them up.


(1, 2)(3, -4) = (1 × 3) + (2 × -4)


⇒ (1, 2)(3, -4) = 3 – 8


⇒ (1, 2)(3, -4) = -5



Similarly, let us fill for the rest of elements.





Let .


Now, compute for DC. [∵ (AB)C = DC]



Multiply 1st row of matrix D by matching members of 1st column of matrix C, then sum them up.


(8, -5)(1, -1) = (8 × 1) + (-5 × -1)


⇒ (8, -5)(1, -1) = 8 + 5


⇒ (8, -5)(1, -1) = 13



Multiply 1st row of matrix D by matching members of 2nd column of matrix C, then sum them up.


(8, -5)(0, 0) = (8 × 0) + (-5 × 0)


⇒ (8, -5)(0, 0) = 0 + 0


⇒ (8, -5)(0, 0) = 0



Similarly, let us fill for the rest of elements.





So,



Take R.H.S: A(BC)


First, compute BC.



Multiply 1st row of matrix B by matching members of 1st column of matrix C, then sum them up.


(2, 3)(1, -1) = (2 × 1) + (3 × -1)


⇒ (2, 3)(1, -1) = 2 – 3


⇒ (2, 3)(1, -1) = -1



Multiply 1st row of matrix B by matching members of 2nd column of matrix C, then sum them up.


(2, 3)(0, 0) = (2 × 0) + (3 × 0)


⇒ (2, 3)(0, 0) = 0 + 0


⇒ (2, 3)(0, 0) = 0



Similarly, let us fill for the rest of the elements.





Let .


Now, compute for AE.



Multiply 1st row of matrix A by matching members of 1st column of matrix E, then sum them up.


(1, 2)(-1, 7) = (1 × -1) + (2 × 7)


⇒ (1, 2)(-1, 7) = -1 + 14


⇒ (1, 2)(-1, 7) = 13



Multiply 1st row of matrix A by matching members of 2nd column of matrix E, then sum them up.


(1, 2)(0, 0) = (1 × 0) + (2 × 0)


⇒ (1, 2)(0, 0) = 0 + 0


⇒ (1, 2)(0, 0) = 0



Similarly, repeat the step for the other elements.





So,



Thus, (AB)C = A(BC).


(ii). We need to verify: A(B + C) = AB + AC


Take L.H.S: A(B + C)


Add B + C.





Let B + C = F, such that



Now, multiply A and F.



Multiply 1st row of matrix A by matching members of 1st column of matrix F, then sum them up.


(1, 2)(3, 2) = (1 × 3) + (2 × 2)


⇒ (1, 2)(3, 2) = 3 + 4


⇒ (1, 2)(3, 2) = 7



Multiply 1st row of matrix A by matching members of 2nd column of matrix F, then sum them up.


(1, 2)(3, -4) = (1 × 3) + (2 × -4)


⇒ (1, 2)(3, -4) = 3 – 8


⇒ (1, 2)(3, -4) = -5



Similarly, repeat the steps for the other elements.





So,



Now, take R.H.S: AB + AC


Compute AB.



Multiply 1st row of matrix A by matching members of 1st column of matrix B, then sum them up.


(1, 2)(2, 3) = (1 × 2) + (2 × 3)


⇒ (1, 2)(2, 3) = 2 + 6


⇒ (1, 2)(2, 3) = 8



Multiply 1st row of matrix A by matching members of 2nd column of matrix B, then sum them up.


(1, 2)(3, -4) = (1 × 3) + (2 × -4)


⇒ (1, 2)(3, -4) = 3 – 8


⇒ (1, 2)(3, -4) = -5



Similarly, repeat the steps for the other elements.





So,



Now, compute AC.



Multiply 1st row of matrix A by matching members of 1st column of matrix C, then sum them up.


(1, 2)(1, -1) = (1 × 1) + (2 × -1)


⇒ (1, 2)(1, -1) = 1 – 2


⇒ (1, 2)(1, -1) = -1



Multiply 1st row of matrix A by matching members of 2nd column of matrix C, then sum them up.


(1, 2)(0, 0) = (1 × 0) + (2 × 0)


⇒ (1, 2)(0, 0) = 0 + 0


⇒ (1, 2)(0, 0) = 0



Similarly, repeat the steps for the other elements.





So,



Adding AB + AC.



Matrices of same order can be added or subtracted.




So, clearly L.H.S = R.H.S.


Thus, A(B + C) = AB + AC.



Question 23.

If , , prove that


Answer:

Given: We have matrices P and Q, such that



To Prove:


Proof: First, we shall compute PQ.



Since, in order to multiply two matrices, A and B, the number of columns in A must equal the number of rows in B. Thus, if A is an m x n matrix and B is an r x s matrix, n = r.


Order of P = 3 × 3


And order of Q = 3 × 3


Number of columns of matrix P = Number of rows of matrix Q = 3


So, P and Q can be multiplied.


So, multiply 1st row of matrix P by matching members of 1st column of matrix Q, then sum them up.


(x, 0, 0)(a, 0, 0) = (x × a) + (0 × 0) + (0 × 0)


⇒ (x, 0, 0)(a, 0, 0) = xa



Multiply 1st row of matrix P by matching members of 2nd column of matrix Q, then sum them up.


(x, 0, 0)(0, b, 0) = (x × 0) + (0 × b) + (0 × 0)


⇒ (x, 0, 0)(0, b, 0) = 0



Similarly, repeat the steps to find other elements.





So,


…(i)


Now, we shall compute QP.



Multiply 1st row of matrix Q by matching members of 1st column of matrix P, then sum them up.


(a, 0, 0)(x, 0, 0) = (a × x) + (0 × 0) + (0 × 0)


⇒ (a, 0, 0)(x, 0, 0) = xa + 0 + 0


⇒ (a, 0, 0)(x, 0, 0) = xa



Similarly, repeat the steps to find other elements.





So,



Thus, .



Question 24.

If , find A.


Answer:

We are given with a matrix equation,


We need to find A.


Take L.H.S:


Let us solve , where




Then,



Order of X = 1 × 3


Order of Y = 3 × 3


Then, resulting order of matrix Z(say) = 1 × 3 [Let Z = XY]


Multiply 1st row of matrix X by matching members of 1st column of matrix Y, then sum them up.


(2, 1, 3)(-1, -1, 0) = (2 × -1) + (1 × -1) + (3 × 0)


⇒ (2, 1, 3)(-1, -1, 0) = -2 – 1 + 0


⇒ (2, 1, 3)(-1, -1, 0) = -3



Multiply 1st row of matrix X by matching members of 2nd column of matrix Y, then sum them up.


(2, 1, 3)(0, 1, 1) = (2 × 0) + (1 × 1) + (3 × 1)


⇒ (2, 1, 3)(0, 1, 1) = 0 + 1 + 3


⇒ (2, 1, 3)(0, 1, 1) = 4



Multiply 1st row of matrix X by matching members of 3rd column of matric Y, then sum them up.


(2, 1, 3)(-1, 0, 1) = (2 × -1) + (1 × 0) + (3 × 1)


⇒ (2, 1, 3)(-1, 0, 1) = -2 + 0 + 3


⇒ (2, 1, 3)(-1, 0, 1) = 1



So,



Now, multiplying Z by .



Order of Z = 1 × 3


Order of Q = 3 × 1


Then, order of the resulting matrix = 1 × 1


Multiply 1st row of matrix Z by matching members of 1st column of matrix Q, then sum them up.


(-3, 4, 1)(1, 0, -1) = (-3 × 1) + (4 × 0) + (1 × -1)


⇒ (-3, 4, 1)(1, 0, -1) = -3 + 0 – 1


⇒ (-3, 4, 1)(1, 0, -1) = -4



Now, since


Thus,


A = [-4]



Question 25.

If , and , verify that A(B + C) = (AB + AC).


Answer:

We are given the matrices A, B and C, such that




We need to verify that, A(B + C) = AB + AC.


Take L.H.S: A(B + C)


Solving (B + C).



These matrices can be added as they have same order.




Now, multiply A by (B + C).


Let (B + C) = D.


We have,


AD = A(B + C)



Order of A = 1 × 2


Order of D = 2 × 3


Then, order of resulting matrix = 1 × 3


Multiply 1st row of matrix A by matching members of 1st column of matrix D, then sum them up.


(2, 1)(4, 9) = (2 × 4) + (1 × 9)


⇒ (2, 1)(4, 9) = 8 + 9


⇒ (2, 1)(4, 9) = 17



Multiply 1st row of matrix A by matching members of 2nd column of matrix D, then sum them up.


(2, 1)(5, 7) = (2 × 5) + (1 × 7)


⇒ (2, 1)(5, 7) = 10 + 7


⇒ (2, 1)(5, 7) = 17



Multiply 1st row of matrix A by matching members of 3rd column of matrix D, then sum them up.


(2, 1)(5, 8) = (2 × 5) + (1 × 8)


⇒ (2, 1)(5, 8) = 10 + 8


⇒ (2, 1)(5, 8) = 18



So,



Now, take R.H.S: AB + AC


Let us compute AB.



Order of A = 1 × 2


Order of B = 2 × 3


Then, order of AB = 1 × 3


Multiply 1st row of matrix A by matching members of 1st column of matrix B, then sum them up.


(2, 1)(5, 8) = (2 × 5) + (1 × 8)


⇒ (2, 1)(5, 8) = 10 + 8


⇒ (2, 1)(5, 8) = 18



Similarly, repeat steps to find the rest of the elements.






Now, let us compute AC.



Order of AC = 1 × 3


Multiply 1st row of matrix A by matching members of 1st column of matrix C, then sum them up.


(2, 1)(-1, 1) = (2 × -1) + (1 × 1)


⇒ (2, 1)(-1, 1) = -2 + 1


⇒ (2, 1)(-1, 1) = -1



Similarly, repeat steps to find the rest of the elements.






Add, AB + AC.





Thus,


A(B + C) = AB + AC.



Question 26.

If , then verify that A2 + A = A(A + I), where I is 3 × 3 unit matrix.


Answer:

We are given with matrix A, such that


We need to verify A2 + A = A(A + I).


Take L.H.S: A2 + A.


Solve for A2.


A2 = A.A



Multiply 1st row of matrix A by matching members of 1st column of matrix A, then sum them up.


(1, 0, -1)(1, 2, 0) = (1 × 1) + (0 × 2) + (-1 × 0)


⇒ (1, 0, -1)(1, 2, 0) = 1 + 0 + 0


⇒ (1, 0, -1)(1, 2, 0) = 1



Similarly, repeat steps to fill for the other elements.





Now, add A2 and A,





Take R.H.S: A(A + I)


First, let us solve for (A + I).





Multiply (A + I) from A.






Since, L.H.S = R.H.S.


Thus, (A2 + A) = A(A + I).



Question 27.

If and , then verify that:

(i) (A’)’ = A

(ii) (AB)’ = B’A’

(iii) (kA)’ = (kA’).


Answer:

We are given with matrices A and B, such that



(i). We need to verify that, (A’)’ = A.


Take L.H.S: (A’)’


In linear algebra, the transpose of a matrix is an operator which flips a matrix over its diagonal, that is it switches the row and column indices of the matrix by producing another matrix denoted as AT or A’.


So, in transpose of a matrix,


Rows of matrix becomes columns of the same matrix.


So,


If ,


(0, -1, 2) and (4, 3, -4) are 1st and 2nd rows respectively, will become 1st and 2nd columns respectively.


Then


Also, if ,


Similarly, (0, 4), (-1, 3) and (2, -4) are 1st, 2nd and 3rd rows respectively, will become 1st, 2nd and 3rd columns respectively.


Then


Note, that


Thus, verified that (A’)’ = A.


(ii). We need to verify that, (AB)’ = B’A’.


Take L.H.S: (AB)’


Compute AB.



Order of A = 2 × 3


Order of B = 3 × 2


Then, order of AB = 2 × 2


Multiplying 1st row of matrix A by matching members of 1st column of matrix B, then sum them up.


(0, -1, 2)(4, 1, 2) = (0 × 4) + (-1 × 1) + (2 × 2)


⇒ (0, -1, 2)(4, 1, 2) = 0 – 1 + 4


⇒ (0, -1, 2)(4, 1, 2) = 3



Similarly, repeat the process to find the other elements.






Transpose of AB is (AB)’.


(3, 9) and (11, -15) are 1st and 2nd rows respectively, will become 1st and 2nd columns respectively.



Take R.H.S: B’A’


If ,


(4, 0), (1, 3) and (2, 6) are 1st, 2nd and 3rd rows of matrix B respectively, will become 1st, 2nd and 3rd columns respectively.



Also, if ,


(0, -1, 2) and (4, 3, -4) are 1st and 2nd rows respectively, will become 1st and 2nd columns respectively.



Multiply B’ by A’.



Order of B’ = 2 × 3


Order of A’ = 3 × 2


Then, order of B’A’ = 2 × 2


Multiply 1st row of matrix B’ by matching members of 1st column of matrix A’, then sum them up.


(4, 1, 2)(0, -1, 2) = (4 × 0) + (1 × -1) + (2 × 2)


⇒ (4, 1, 2)(0, -1, 2) = 0 – 1 + 4


⇒ (4, 1, 2)(0, -1, 2) = 3



Similarly, repeat the same steps to find out other elements.






Since, L.H.S = R.H.S.


Thus, (AB)’ = B’A’.


(iii). We need to verify that, (kA)’ = kA’.


Take L.H.S: (kA)’


We know that,



Multiply k on both sides, (k is a scalar quantity)





Now, to find transpose of kA,


(0, -k, 2k) and (4k, 3k, -4k) are 1st and 2nd rows of matrix kA respectively, will become 1st and 2nd columns respectively.



Take R.H.S: kA’


If


Then, for transpose of A,


(0, -1, 2) and (4, 3, -4) are 1st and 2nd rows of matrix A respectively, will become 1st and 2nd columns respectively.



Multiply k on both sides,





Note that, L.H.S = R.H.S.


Thus, (kA)’ = kA’.



Question 28.

If then verify that:

(i) (2A + B)’ = 2A’ + B’

(ii) (A – B)’ = A’ – B’.


Answer:

We are given matrices A and B, such that



In linear algebra, the transpose of a matrix is an operator which flips a matrix over its diagonal, that is it switches the row and column indices of the matrix by producing another matrix denoted as AT or A’.


So, in transpose of a matrix,


Rows of matrix becomes columns of the same matrix.


(i). We need to verify that, (2A + B)’ = 2A’ + B’.


Take L.H.S: (2A + B)’


Substitute the matrices A and B, in (2A + B)’.







For transpose of (2A + B),


(3, 6), (14, 6) and (17, 15) are 1st, 2nd and 3rd rows respectively, will become 1st, 2nd and 3rd columns respectively.



Take R.H.S: 2A’ + B’


If ,


(1, 2), (4, 1) and (5, 6) are 1st, 2nd and 3rd rows respectively, will become 1st, 2nd and 3rd columns respectively.



Multiply both sides by 2,





Also,


If .


(1, 2), (6, 4) and (7, 3) are 1st, 2nd and 3rd rows respectively, will become 1st, 2nd and 3rd columns respectively.



Now, add 2A’ and B’.





Since, L.H.S = R.H.S


Thus, (2A + B)’ = 2A’ + B’.


(ii). We need to verify that, (A – B)’ = A’ – B’.


Take L.H.S: (A – B)’


Substitute the matrices A and B in (A – B)’.





To find transpose of (A – B),


(0, 0), (-2, -3) and (-2, 3) are 1st, 2nd and 3rd rows respectively, will become 1st, 2nd and 3rd columns respectively.



Take R.H.S: A’ – B’


If ,


(1, 2), (4, 1) and (5, 6) are 1st, 2nd and 3rd rows respectively, will become 1st, 2nd and 3rd columns respectively.



Also,


If ,


(1, 2), (6, 4) and (7, 3) are 1st, 2nd and 3rd rows respectively, will become 1st, 2nd and 3rd columns respectively.



Subtract B’ from A’,





Since, L.H.S = R.H.S


Thus, (A – B)’ = A’ – B’.



Question 29.

Show that A’A and AA’ are both symmetric matrices for any matrix A.


Answer:

We must understand,

In linear algebra, a symmetric matrix is a square matrix that is equal to its transpose. Formally, because equal matrices have equal dimensions, only square matrices can be symmetric.


And we know that, transpose of AB is given by


(AB)’ = B’A’


Using this result, take transpose of A’A.


Transpose of A’A = (A’A)T = (A’A)’


Using, transpose of A’A = (A’A)’


⇒ (A’A)’ = A’(A’)’


And also,


(A’)’ = A


So,


(A’A)’ = A’A


Since, (A’A)’ = A’A


This means, A’A is symmetric matrix for any matrix A.


Now, take transpose of AA’.


Transpose of AA’ = (AA’)’


⇒ (AA’)’ = (A’)’A’ [∵ (AB)’ = B’A’]


⇒ (AA’)’ = AA’ [∵ (A’)’ = A]


Since, (AA’)’ = AA’


This means, AA’ is symmetric matrix for any matrix A.


Thus, A’A and AA’ are symmetric matrix for any matrix A.



Question 30.

Let A and B be square matrices of the order 3 × 3. Is (AB)2 = A2B2 ? Give reasons.


Answer:

We are given that,

A and B are square matrices of the order 3 × 3.


We need to check whether (AB)2 = A2B2 is true or not.


Take (AB)2.


(AB)2 = (AB)(AB)


[∵ A and B are of order (3 × 3) each, A and B can be multiplied; A and B be any matrices of order (3 × 3)]


⇒ (AB)2 = ABAB


[∵ (AB)(AB) = ABAB]


⇒ (AB)2 = AABB


[∵ ABAB = AABB; as A can be multiplied with itself and B can be multiplied by itself]


⇒ (AB)2 = A2B2


So, note that, (AB)2 = A2B2 is possible.


But this is possible if and only if BA = AB.


And BA = AB is always true whenever A and B are square matrices of any order. And for BA = AB,


(AB)2 = A2B2



Question 31.

Show that if A and B are square matrices such that AB = BA, then (A + B)2 = A2 + 2AB + B2.


Answer:

By matrix multiplication we can write:


(A + B)2 = (A+B)(A+B) = A2 + AB + BA + B2


We know that matrix multiplication is not commutative but it is given that : AB = BA


∴ (A + B)2 = A2 + AB + AB + B2


⇒ (A + B)2 = A2 + 2AB + B2 …proved



Question 32.

Let and a = 4, b = –2.

Show that:

A + (B + C) = (A + B) + C


Answer:

Given A = B = and C =


LHS = A + (B + C) =


⇒ LHS =


⇒ LHS =


RHS = (A + B) + C =


⇒ RHS =


⇒ RHS =


Clearly LHS = RHS =


Hence,


A + (B + C) = (A + B) + C …proved



Question 33.

Let and a = 4, b = –2.

Show that:

A(BC) = (AB)C


Answer:

To prove: A(BC) = (AB)C


LHS = A(BC) =


⇒ LHS =


⇒ LHS =


⇒ LHS =


RHS = (AB)C =


Performing matrix multiplication as done for LHS


⇒ RHS =


⇒ RHS =


Clearly LHS = RHS =


∴ A(BC) = (AB)C …proved



Question 34.

Let and a = 4, b = –2.

Show that:

(a + b)B = aB + bB


Answer:

To prove: (a + b)B = aB + bB


Given, a = 4 and b = -2


LHS = (4+(-2))B =


RHS = aB + bB =


⇒ RHS =


Clearly LHS = RHS =


Hence,


(a + b)B = aB + bB …proved



Question 35.

Let and a = 4, b = –2.

Show that:

a(C – A) = aC –aA


Answer:

To prove: a(C – A) = aC –aA


As, LHS = a(C – A) = 4


⇒ LHS = 4


RHS = aC – aA =


⇒ aC – aA =


Clearly LHS = RHS =


Hence,


a(C – A) = aC –aA …proved



Question 36.

Let and a = 4, b = –2.

Show that:

(AT)T = A


Answer:

To prove: (AT)T = A


As transpose of a matrix is obtained by interchanging rows with respective columns.


LHS = (AT)T = = RHS


Hence, proved.



Question 37.

Let and a = 4, b = –2.

Show that:

(bA)T = bAT


Answer:

a) To prove: (bA)T = bAT


As, LHS = (bA)T = (-2A)T


⇒ LHS =


Similarly,


RHS =


Clearly LHS = RHS =


Hence, (bA)T = bAT …proved



Question 38.

Let and a = 4, b = –2.

Show that:

(AB)T = BT AT


Answer:

b) To prove: (AB)T = BTAT


Clearly, LHS = (AB)T =


First multiplying the matrix and then taking the transpose.


∴ LHS =


⇒ LHS =


∴ LHS =


As RHS = BTAT


We will first take transpose of matrices and then multiply


RHS =


⇒ RHS =


Clearly, LHS = RHS =


Hence (AB)T = BTAT …proved



Question 39.

Let and a = 4, b = –2.

Show that:

(A – B)C = AC – BC


Answer:

c) To prove: (A – B)C = AC – BC


As, LHS = (A – B)C


Putting the values of A,B and C and multiplying by rule of matrix multiplication.


LHS =


⇒ LHS =


RHS = AC – BC =


⇒ RHS =


Clearly, LHS = RHS =


Hence (A-B)C = AC - BC…proved



Question 40.

Let and a = 4, b = –2.

Show that:

(A – B)T = AT – BT


Answer:

To Prove: (A – B)T = AT - BT


LHS = (A – B)T =


RHS = AT – BT =


∴ RHS =


Clearly, LHS = RHS =


Hence (A-B)T = AT – BT …proved



Question 41.

If then show that


Answer:

As A =


∴ A2 =


By matrix multiplication:


A2 =


⇒ A2 =


As we know that:


2 sin θ cos θ = sin 2θ and cos2 θ – sin2 θ = cos 2θ


∴ A2 = …Hence proved



Question 42.

If and x2 = –1, then show that (A + B)2 = A2 + B2.


Answer:

As, LHS = (A + B)2 =


⇒ LHS =


By matrix multiplication we can write LHS as –


LHS =


⇒ LHS =


Given x2 = -1


∴ LHS =


RHS = A2 + B2 =


⇒ RHS =


By matrix multiplication we can write-


RHS =


Given x2 = -1


∴ RHS =


Clearly RHS = LHS =


Hence, (A + B)2 = A2 + B2 …proved



Question 43.

Verify that A2 = I when


Answer:

We need to prove that: A2 = I =


∵ A =


∴ A2 =


By matrix multiplication we have-


A2 =


⇒ A2 =


∴ A2 =


Hence Verified



Question 44.

Prove by Mathematical Induction that (A’)n = (An)’, where n ∈ N for any square matrix A.


Answer:

By principle of mathematical induction we say that if a statement P(n) is true for n = 1 and if we assume P(k) to be true for some random natural number k and usnig it if we prove P(k+1) to be true we can say that P(n) is true for all natural numbers.

We are given to prove that (A’)n = (An)’.


Let P(n) be the statement : (A’)n = (An)’.


Clearly, P(1): (A’)1 = (A1)’


⇒ P(1) : A’ = A’


⇒ P(1) is true


Let P(k) be true.


∴ (A’)k = (Ak)’ …(1)


Let’s take P(k+1) now:


∵ (Ak+1)’ = (AkA)’


We know that by properties of transpose of a matrix:


(AB)T = BTAT


∴ (AkA)’ = A’(Ak)’ = A’(A’)k = (A’)k+1


Thus, (Ak+1)’ = (A’)k+1


∴ P(k+1) is true.


Hence,


We can say that: (A’)n = (An)’ is true for all n ∈ N.



Question 45.

Find inverse, by elementary row operations (if possible), of the following matrices.



Answer:

Let A =


To apply elementary row transformations we write:


A = IA where I is the identity matrix


We proceed with operations in such a way that LHS becomes I and the transformations in I give us a new matrix such that


I = XA


And this X is called inverse of A = A-1


So we have:



Applying R2→ R2 + 5R1



Applying R2→ (1/22)R2



Applying R1→ R1 – 3R2



As we got Identity matrix in LHS.


∴ A-1 =



Question 46.

Find inverse, by elementary row operations (if possible), of the following matrices.



Answer:

Let B =


To apply elementary row transformations we write:


B = IB where I is the identity matrix


We proceed with operations in such a way that LHS becomes I and the transformations in I give us a new matrix such that


I = XB


And this X is called inverse of B = B-1


So we have:



Applying R2→ R2 + 2R1



As we got all zeroes in one of the row of matrix in LHS.


So by any means we can make identity matrix in LHS.


∴ inverse of B does not exist.


B-1 does not exist. …ans



Question 47.

If then find values of x, y, z and w.


Answer:

Given,



As the 2 matrices are equal. So corresponding elements of both the matrix must also hold the equality.


∴ xy = 8 ; w = 4 ; z + 6 = 0 and x + y = 6


Hence, we get:


w = 4


z = -6


∵ x + y = 6


⇒ y = 6 – x


∴ x(6-x) = 8


⇒ x2 – 6x + 8 = 0


⇒ x2 – 4x – 2x + 8 = 0


⇒ x(x – 4) – 2(x – 4) = 0


⇒ (x – 2)(x – 4) = 0


⇒ x = 2 or x = 4


When x = 2 ; y = 4


And when x = 4 ; y = 2


Thus,


x = 2 or 4 ; y = 4 or 2 ; z = -6 and w = 4 …ans



Question 48.

If find a matrix C such that 3A + 5B + 2C is a null matrix.


Answer:

Given that:


3A + 5B + 2C = O = null matrix


C = ?


As,




∴ 2C =


⇒ 2C =


∴ C = …ans



Question 49.

If then find A2 – 5A – 14I. Hence, obtain A3.


Answer:

Given, A =


∴ A2 =


By matrix multiplication we can write:


A2 =


⇒ A2 = …(1)


As we have to find: A2 – 5A – 14I


∴ A2 – 5A – 14I =


⇒ A2 – 5A – 14I =


⇒ A2 – 5A – 14I =


⇒ A2 – 5A – 14I = = O


We need to find value of A3 using the above equation:


Now we have,


A2 – 5A – 14I = O


⇒ A2 = 5A + 14I


Multiplying with A both sides


⇒ A2.A = 5A.A + 14IA


⇒ A3 = 5A2 + 14A


Using equation 1 we get:


⇒ A3 =


⇒ A3 =


⇒ A3 =



Question 50.

Find the value of a, b, c and d, if


Answer:

Given,



We need to find the value of a, b, c and d.


As,



As both matrices are equal so their corresponding elements must also be equal.


∴ 3a = a + 4


⇒ 2a = 4


⇒ a = 2


Similarly,


3b = 6 + a + b


⇒ 2b = 6 + a


As from above a = 2


∴ 2b = 6+2 = 8


⇒ b = 4


Also 3d = 2d + 3


⇒ d = 3


And,


3c = -1 + c + d


⇒ 2c = d – 1


⇒ 2c = 3-1


⇒ c = 2/2 = 1


Thus a = 2, b = 4, c = 1 and d = 3.



Question 51.

Find the matrix A such that



Answer:

Given,



As A is multiplied with a matrix of order 3×2 and gives a resultant matrix of order 3×3


For matrix multiplication to be possible A must have 2 rows and as resultant matrix is of 3rd order A must have 3 columns


∴ A is matrix of order 2×3


Let A = where a, b, c, d, e and f are unknown variables.



∴ By matrix multiplication we have-



By equating the elements of 2 equal matrices we get-


a = 1 ; b = -2 and c = -5


also,


2a – d = -1 ⇒ d = 2a + 1 = 2 + 1 = 3


∴ d = 3


2b – e = -8 ⇒ e = 2b + 8 = -4 + 8 = 4


∴ e = 4


Similarly, f = 2c + 10 = 0


∴ A =



Question 52.

If find A2 + 2A + 7I


Answer:

Given,



∵ A2 = A.A


⇒ A2 =


By matrix multiplication, we get


A2 =


⇒ A2 =


∴ A2 + 2A + 7I =


⇒ A2 + 2A + 7I =


⇒ A2 + 2A + 7I =


⇒ A2 + 2A + 7I = …ans



Question 53.

If and A–1 = A’, find value of α.


Answer:

Given, A =


We know that transpose of a matrix is obtained by interchanging rows with respective columns.


∴ A’ =


Inverse of a matrix A = A-1 =


Clearly |A| =


∴ |A| = {using trigonometric identity}


Adj(A) is given by the transpose of the cofactor matrix.


∴ adj(A) =


∴ A-1 =


According to question:


A’ = A-1



As both the matrices are equal irrespective of the value of α.


∴ α can be any real number …ans



Question 54.

If the matrix is a skew symmetric matrix, find the values of a, b and c.


Answer:

A matrix is said to be skew-symmetric if A = -A’


Let, A =


As, A is skew symmetric matrix.


∴ A = -A’





Equating the respective elements of both matrices we get-


a = -2 ; c = -3 ; b = -b ⇒ 2b = 0 ⇒ b = 0


Thus, we have-


a = -2 , b = 0 and c = -3 …ans



Question 55.

If then show that

P(x).P(y) = P(x + y) = P(y).P(x)


Answer:

Given,


P(x) = …(1)


P(y) =


∴ P(x).P(y) =


⇒ P(x).P(y) =


We know that-


cos x cos y + sin x sin y = cos (x – y)


cos x sin y + sin x cos y = sin (x + y)


and cos x cos y – sin x sin y = cos (x + y)


⇒ P(x).P(y) =


In comparison with equation 1 we can say that:


…(2)


∴ P(x).P(y) = P(x + y)


Similarly, we can show for P(y).P(x):


P(y).P(x) =


By matrix multiplication, we have –


P(y).P(x) =


⇒ P(y).P(x) =


⇒ P(y).P(x) = …(3)


∴ From equation 2 and 3:


P(x).P(y) = P(y).P(x) = P(x + y) …ans



Question 56.

If A is square matrix such that A2 = A, show that (I + A)3 = 7A + I.


Answer:

Given that,


A2 = A


∵ (a+b)3 = a3 + b3 + 3a2b + 3ab2


As, (I + A)3 = I3 + A3 + 3I2A + 3IA2


∵ I is an identity matrix.


∴ I3 = I2 = I


∴ (I + A)3 = I + A3 + 3IA + 3IA


As, I is an identity matrix.


∴ IA = AI = A


⇒ (I + A)3 = I + A3 + 6IA


∵ A2 = A


⇒ (I + A)3 = I + A2.A + 6A


⇒ (I + A)3 = I + A.A + 6A


⇒ (I + A)3 = I + A2 + 6A


⇒ (I + A)3 = I + A + 6A = I + 7A


Hence,


(I + A)3 = I + 7A …proved



Question 57.

If A, B are square matrices of same order and B is a skew-symmetric matrix, show that A’ BA is skew symmetric.


Answer:

A matrix is said to be skew-symmetric if A = -A’


Given, B is a skew-symmetric matrix.


∴ B = -B’


Let C = A’ BA …(1)


We have to prove C is skew-symmetric.


To prove: C = -C’


As C’ = (A’BA)’


We know that: (AB)’ = B’A’


⇒ C’ = (A’BA)’ = A’B’(A’)’


⇒ C’ = A’B’A {∵ (A’)’ = A}


⇒ C’ = A’(-B)A


⇒ C’ = -A’BA …(2)


From equation 1 and 2:


We have,


C’ = -C


Thus we say that C = A’ BA is a skew-symmetric matrix.



Question 58.

If AB = BA for any two square matrices, prove by mathematical induction that (AB)n = An Bn.


Answer:

By principle of mathematical induction we say that if a statement P(n) is true for n = 1 and if we assume P(k) to be true for some random natural number k and usnig it if we prove P(k+1) to be true we can say that P(n) is true for all natural numbers.

We are given to prove that (AB)n = AnBn


Let P(n) be the statement : (AB)n = AnBn


Clearly, P(1): (AB)1 = A1B1


⇒ P(1) : AB = AB


⇒ P(1) is true


Let P(k) be true.


∴ (AB)k = AkBk …(1)


Let’s take P(k+1) now:


∵ (AB)k+1 = (AB)k(AB)


⇒ (AB)k+1 = AkBk(AB)


NOTE: As we know that Matrix multiplication is not commutative. So we can’t write directly that AkBk(AB) = Ak+1Bk+1


But we are given that AB = BA


∴ (AB)k+1 = AkBk(AB)


⇒ (AB)k+1 = AkBk-1(BAB)


As AB = BA


⇒ (AB)k+1 = AkBk-1(ABB)


⇒ (AB)k+1 = AkBk-1(AB2)


⇒ (AB)k+1 = AkBk-2(BAB2)


⇒ (AB)k+1 = AkBk-2(ABB2)


⇒ (AB)k+1 = AkBk-2(AB3)


We observe that one power of B is decreasing while other is increasing. After certain repetitions decreasing power of B will become I


And at last step:


⇒ (AB)k+1 = AkI(ABk+1)


⇒ (AB)k+1 = AkABk+1


⇒ (AB)k+1 = Ak+1Bk+1


Thus P(k+1) is true when P(k) is true.


∴ (AB)n = An Bn ∀ n ∈ N when AB = BA.



Question 59.

Find x, y, z if satisfies A’ = A–1


Answer:

Given,



We need to find x, y and z such that A’ = A-1


If A’ = A-1


Pre-multiplying A on both sides:


AA’ = AA-1


⇒ AA’ = I where I is the identity matrix.




By matrix multiplication we have:



On equating the corresponding elements of matrix.


We need basically 3 equations as we have 3 variables to solve for. You can pick any three elements and equate them.


We have:


4y2 + z2 = 1 …(1)


x2 + y2 + z2 = 1 …(2)


2y2 – z2 = 0 …(3)


Adding equation 2 and 3:


6y2 = 1


⇒ y2 = 1/6



From equation 3:


Z2 = 2y2


⇒ z2 = 2(1/6)


∴ z2 = 1/3



From equation 2:


x2 = 1 – y2 – z2


⇒ x2 = 1 – (1/6) – (1/3)


⇒ x2 = 1 – 1/2 = 1/2



Thus,


; and



Question 60.

If possible, using elementary row transformations, find the inverse of the following matrices



Answer:

Let A =


To apply elementary row transformations we write:


A = IA where I is the identity matrix


We proceed with operations in such a way that LHS becomes I and the transformations in I give us a new matrix such that


I = XA


And this X is called inverse of A = A-1


Note: Never apply row and column transformations simultaneously over a matrix.


So we have:



Applying R2→ R2 + R1


=


Applying R3→ R3 - R2


=


Applying R1→ R1 + R2


=


Applying R2→ R2 - 3R1


=


Applying R3→ (-1)R3


=


Applying R1→ R1 + 10R3 and R2→ R2 + 17R3


=


Applying R1→ (-1)R1 and R2→ (-1)R2


=


As we got Identity matrix in LHS.


∴ A-1 =



Question 61.

If possible, using elementary row transformations, find the inverse of the following matrices



Answer:

Let A =


To apply elementary row transformations we write:


A = IA where I is the identity matrix


We proceed with operations in such a way that LHS becomes I and the transformations in I give us a new matrix such that


I = XA


And this X is called inverse of A = A-1


Note: Never apply row and column transformations simultaneously over a matrix.


So we have:



Applying R2→ R2 + R3



Applying R1→ R1 - 2R3



Applying R2→ R1 + R2



As second row of LHS contains all zeros, So by anyhow we are never going to get Identity matrix in LHS.


∴ Inverse of A does not exist.


A-1 does not exist. …ans



Question 62.

If possible, using elementary row transformations, find the inverse of the following matrices



Answer:

Let A =


To apply elementary row transformations we write:


A = IA where I is the identity matrix


We proceed with operations in such a way that LHS becomes I and the transformations in I give us a new matrix such that


I = XA


And this X is called inverse of A = A-1


Note: Never apply row and column transformations simultaneously over a matrix.


So we have:



Applying R2→ R2 – (5/2)R1



Applying R3→ R3 - R2



Applying R1→ R1 + R2


=


Applying R2→ R2 - 5R3


=


Applying R1→ R1 + 2R3


=


Applying R1→ (1/2)R1 and R3→ 2R3


=


As we got Identity matrix in LHS.


∴ A-1 =



Question 63.

The matrix is a
A. square matrix

B. diagonal matrix

C. unit matrix

D. none


Answer:

As P has equal number of rows and columns and thus it matches with the definition of square matrix.


The given matrix does not satisfy the definition of unit and diagonal matrices.


∴ Option (A) is the only correct answer.


Question 64.

Total number of possible matrices of order 3 × 3 with each entry 2 or 0 is
A. 9

B. 27

C. 81

D. 512


Answer:

As matrix has total 3× 3 = 9 elements.


As each element can take 2 values (0 or 2)


∴ By simple counting principle we can say that total number of possible matrices = total number of ways in which 9 elements can take possible values = 29 = 512


Clearly it matches with option D.


∴ option (D) is the only correct answer.


Question 65.

If then the value of x + y is
A. x = 3, y = 1

B. x = 2, y = 3

C. x = 2, y = 4

D. x = 3, y = 3


Answer:

Given,



By equality of two matrices, we have-


4x = x + 6


⇒ 3x = 6 ⇒ x = 2


Also, 2x + y = 7


⇒ y = 7 – 2x = 7 – 4 = 3


∴ y = 3


As only option (B) matches with our answer.


∴ Option(B) is the correct answer.


Question 66.

If then A – B is equal to
A. I

B. O

C. 2I

D.


Answer:

A very basic idea of Inverse trigonometric function is required to solve the problem.

cos-1 x + sin-1 x = π/2 and cot-1 x + tan-1 x = π/2


As,


And


∴ A – B =


⇒ A – B =


∴ A – B =


Clearly It matches with option (D)


∴ option(D) is the only correct answer.


Question 67.

If A and B are two matrices of the order 3 × m and 3 × n, respectively, and m = n, then the order of matrix (5A – 2B) is
A. m × 3

B. 3 × 3

C. m × n

D. 3 × n


Answer:

As order of A is 3 × m and order of B is 3 × n


As m = n. So, order of A and B is same = 3 × m


∴ subtraction is possible.


And (5A – 3B) also has same order.


Question 68.

If then A2 is equal to
A.

B.

C.

D.


Answer:

Let A =


∴ A2 =


By matrix multiplication:


⇒ A2 = which matches with option (D)


∴ Option (D) is the correct answer.


Question 69.

If matrix , where aij = 1 if i ≠ j

aij = 0 if i = j, then A2 is equal to
A. I

B. A

C. 0

D. None of these


Answer:

According to question:


a11 = 0 , a12 = 1 , a21 = 1 and a22 = 0


∴ A =


∴ A2 =


By matrix multiplication:


⇒ A2 = which matches with option (A)


∴ Option (A) is the correct answer.


Question 70.

The matrix is a
A. identity matrix

B. symmetric matrix

C. skew symmetric matrix

D. none of these


Answer:

Let A =


Clearly,


A’ =


As, AT = A


∴ It is symmetric matrix.


∴ Option(B) is the correct answer.


Question 71.

The matrix is a
A. diagonal matrix

B. symmetric matrix

C. skew symmetric matrix

D. scalar matrix


Answer:

Let A =


Clearly,


A’ =


As, AT = -A


∴ It is skew - symmetric matrix.


∴ Option(C) is the correct answer.


Question 72.

If A is matrix of order m × n and B is a matrix such that AB’ and B’A are both defined, then order of matrix B is
A. m × m

B. n × n

C. n × m

D. m × n


Answer:

As AB’ is defined. So, B’ must have n rows.


∴ B has n columns.


And, B’A is also defined. As, A’ has order n × m


∴ B’A to exist B must have m rows.


∴ m × n is the order of B.


Option (D) is the correct answer.


Question 73.

If A and B are matrices of same order, then (AB’ – BA’) is a
A. skew symmetric matrix

B. null matrix

C. symmetric matrix

D. unit matrix


Answer:

Let C = (AB’ – BA’)


C’ = (AB’ – BA’)’


⇒ C’ = (AB’)’ – (BA’)’


⇒ C’ = (B’)’A’ – (A’)’B’


⇒ C’ = BA’ – AB’


⇒ C’ = -C


∴ C is a skew-symmetric matrix.


Clearly Option (A) matches with our deduction.


∴ Option (A) is the correct.


Question 74.

If A is a square matrix such that A2 = I, then (A – I)3 + (A + I)3 – 7A is equal to
A. A

B. I – A

C. I + A

D. 3A


Answer:

As, (A – I)3 + (A + I)3


Use a3 + b3 = (a + b)(a2 + ab + b2)


Also A2 = I


∴ then (A – I)3 + (A + I)3 – 7A = 2A + 6A – 7A = A


Clearly our answer matches with option (A)


∴ option (A) is the correct answer.


Question 75.

For any two matrices A and B, we have
A. AB = BA

B. AB ≠ BA

C. AB = O

D. None of the above


Answer:

For any two matrix:


Not always option A , B and C are true.


∴ Option (D) is the only suitable answer


Question 76.

On using elementary column operations C2→ C2 — 2C1 in the following matrix equation

we have:
A.

B.

C.

D.


Answer:


For column transformation, we operate the post matrix.


As,



Applying C2→ C2 — 2C1



Clearly, it matches with option (D).


∴ Option (D) is the correct answer.


Question 77.

On using elementary row operation R1→ R1 — 3R2 in the following matrix equation:

we have:
A.

B.

C.

D.


Answer:

Elementary row transformation is applied on the first matrix of RHS.



Applying R1→ R1 — 3R2 we have -




Clearly it matches with option (A)


∴ Option (A) is the correct answer.


Question 78.

Fill in the blanks in each of the

______ matrix is both symmetric and skew symmetric matrix.


Answer:

A Zero matrix

∴ Let A be the symmetric and skew symmetric matrix.


⇒ A’=A (Symmetric)


⇒ A’=-A (Skew-Symmetric)


Considering the above two equations,


⇒ A=-A


⇒ 2A=0


⇒ A=0 (A Zero Matrix)


Hence Zero matrix is both symmetric and skew symmetric matrix.



Question 79.

Fill in the blanks in each of the

______ matrix is both symmetric and skew symmetric matrix.


Answer:

A Zero matrix

∴ Let A be the symmetric and skew symmetric matrix.


⇒ A’=A (Symmetric)


⇒ A’=-A (Skew-Symmetric)


Considering the above two equations,


⇒ A=-A


⇒ 2A=0


⇒ A=0 (A Zero Matrix)


Hence Zero matrix is both symmetric and skew symmetric matrix.



Question 80.

Fill in the blanks in each of the

Sum of two skew symmetric matrices is always _______ matrix.


Answer:

A skew symmetric matrix

∴ Let A and B are two skew symmetric matrices.


⇒ A’=-A ..(1)


⇒ B’=-B ..(2)


Now Let A+B=C ..(3)


⇒ C’=(A+B)’=A’+B’


⇒ A’+B’=(-A)+(-B)


⇒ (-A)+(-B)=-(A+B)=-C


⇒ C’=-C (Skew Symmetric matrix)



Question 81.

Fill in the blanks in each of the

Sum of two skew symmetric matrices is always _______ matrix.


Answer:

A skew symmetric matrix

∴ Let A and B are two skew symmetric matrices.


⇒ A’=-A ..(1)


⇒ B’=-B ..(2)


Now Let A+B=C ..(3)


⇒ C’=(A+B)’=A’+B’


⇒ A’+B’=(-A)+(-B)


⇒ (-A)+(-B)=-(A+B)=-C


⇒ C’=-C (Skew Symmetric matrix)



Question 82.

Fill in the blanks in each of the

The negative of a matrix is obtained by multiplying it by ________.


Answer:

-1

The negative of a matrix is obtained by multiplying it by -1.


For example:


Let A =


So


= - A



Question 83.

Fill in the blanks in each of the

The negative of a matrix is obtained by multiplying it by ________.


Answer:

-1

The negative of a matrix is obtained by multiplying it by -1.


For example:


Let A =


So


= - A



Question 84.

Fill in the blanks in each of the

The product of any matrix by the scalar _____ is the null matrix.


Answer:

The null matrix is the one in which all elements are zero.

If we want to make A = a null matrix we need to multiply it by 0.


0A =



The product of any matrix by the scalar 0 is the null matrix.



Question 85.

Fill in the blanks in each of the

A matrix which is not a square matrix is called a _____ matrix.


Answer:

Rectangular Matrix

As a square matrix is the one in which there are same number of rows and columns.


Eg: A =


Here there are 2 rows and 2 columns.


The matrix which is not square is called rectangular matrix as it does not have same number of rows and columns.


Eg


Here number of rows are 2 and columns are 3.



Question 86.

Fill in the blanks in each of the

A matrix which is not a square matrix is called a _____ matrix.


Answer:

Rectangular Matrix

As a square matrix is the one in which there are same number of rows and columns.


Eg: A =


Here there are 2 rows and 2 columns.


The matrix which is not square is called rectangular matrix as it does not have same number of rows and columns.


Eg


Here number of rows are 2 and columns are 3.



Question 87.

Fill in the blanks in each of the

Matrix multiplication is _____ over addition.


Answer:

Distributive

⇒ Matrix multiplication is distributive over addition.


i.e A(B+C)=AB+AC


and (A+B)C=AC+BC



Question 88.

Fill in the blanks in each of the

Matrix multiplication is _____ over addition.


Answer:

Distributive

⇒ Matrix multiplication is distributive over addition.


i.e A(B+C)=AB+AC


and (A+B)C=AC+BC



Question 89.

Fill in the blanks in each of the

If A is a symmetric matrix, then A3 is a ______ matrix.


Answer:

A3 is Also a symmetric matrix.

Given: A’=A ..(1)


⇒ (A2)’=(AA)’=A’A’


⇒ A’A’=(A)(A)=A2


⇒ (A2)’=A2 (symmetric matrix) ..(2)


⇒ (A3)’=(A(A2))’=(A2)’A’


⇒ (A2)’A’=A2A= A3 (Using (1) and (2) )


⇒ (A3)’=A3 (symmetric matrix)



Question 90.

Fill in the blanks in each of the

If A is a symmetric matrix, then A3 is a ______ matrix.


Answer:

A3 is Also a symmetric matrix.

Given: A’=A ..(1)


⇒ (A2)’=(AA)’=A’A’


⇒ A’A’=(A)(A)=A2


⇒ (A2)’=A2 (symmetric matrix) ..(2)


⇒ (A3)’=(A(A2))’=(A2)’A’


⇒ (A2)’A’=A2A= A3 (Using (1) and (2) )


⇒ (A3)’=A3 (symmetric matrix)



Question 91.

Fill in the blanks in each of the

If A is a skew symmetric matrix, then A2 is a _________.


Answer:

A2 is a symmetric matrix.

Given: A’=-A


⇒ (A2)’=(AA)’=A’A’


⇒ A’A’=(-A)(-A)=A2


⇒ (A2)’=A2 (symmetric matrix)



Question 92.

Fill in the blanks in each of the

If A is a skew symmetric matrix, then A2 is a _________.


Answer:

A2 is a symmetric matrix.

Given: A’=-A


⇒ (A2)’=(AA)’=A’A’


⇒ A’A’=(-A)(-A)=A2


⇒ (A2)’=A2 (symmetric matrix)



Question 93.

Fill in the blanks in each of the

If A and B are square matrices of the same order, then

(i) (AB)’ = ________.

(ii) (kA)’ = ________. (k is any scalar)

(iii) [k (A – B)]’ = ________.


Answer:

(i) (AB)’ = ________.


(AB)’ = B’A’


Let A be matrix of order m× n and B be of n× p.


A is of order n× m and B is of order p× n.


Hence B A is of order p× m.


So, AB is of order m× p.


And (AB) is of order p× m.


We can see (AB) and B A are of same order p× m.


Hence (AB) = B A


Hence proved.


(ii) (kA)’ = ________. (k is any scalar)


If a scalar “k” is multiplied to any matrix the new matrix becomes


K times of the old matrix.


Eg: A =


2A =


=


(2A) =


A =


Now 2A =


=


Hence (2A) =2A


Hence (kA)’ = k(A)’


(iii) [k (A – B)]’ = ________.


A =


A =


2A = 2


=


B=


B =


2B =


=


A-B =


Now Let k =2


2(A-B) =


=


[2(A-B)] =


2A – 2B =


=


A – B =


=


2(A – B) = 2


=


Hence we can see [k (A – B)]’= k(A)’- k(B)’= k(A’-B’)



Question 94.

Fill in the blanks in each of the

If A and B are square matrices of the same order, then

(i) (AB)’ = ________.

(ii) (kA)’ = ________. (k is any scalar)

(iii) [k (A – B)]’ = ________.


Answer:

(i) (AB)’ = ________.


(AB)’ = B’A’


Let A be matrix of order m× n and B be of n× p.


A is of order n× m and B is of order p× n.


Hence B A is of order p× m.


So, AB is of order m× p.


And (AB) is of order p× m.


We can see (AB) and B A are of same order p× m.


Hence (AB) = B A


Hence proved.


(ii) (kA)’ = ________. (k is any scalar)


If a scalar “k” is multiplied to any matrix the new matrix becomes


K times of the old matrix.


Eg: A =


2A =


=


(2A) =


A =


Now 2A =


=


Hence (2A) =2A


Hence (kA)’ = k(A)’


(iii) [k (A – B)]’ = ________.


A =


A =


2A = 2


=


B=


B =


2B =


=


A-B =


Now Let k =2


2(A-B) =


=


[2(A-B)] =


2A – 2B =


=


A – B =


=


2(A – B) = 2


=


Hence we can see [k (A – B)]’= k(A)’- k(B)’= k(A’-B’)



Question 95.

Fill in the blanks in each of the

If A is skew symmetric, then kA is a ______. (k is any scalar)


Answer:

A skew symmetric matrix.

Given A’=-A


⇒ (kA)’=k(A)’=k(-A)


⇒ (kA)’=-(kA)



Question 96.

Fill in the blanks in each of the

If A is skew symmetric, then kA is a ______. (k is any scalar)


Answer:

A skew symmetric matrix.

Given A’=-A


⇒ (kA)’=k(A)’=k(-A)


⇒ (kA)’=-(kA)



Question 97.

Fill in the blanks in each of the

If A and B are symmetric matrices, then

(i) AB – BA is a _________.

(ii) BA – 2AB is a _________.


Answer:

(i) AB – BA is a Skew Symmetric matrix


Given A’=A and B’=B


⇒ (AB-BA)’=(AB)’-(BA)’


⇒ (AB)’-(BA)’=B’A’-A’B’


⇒ B’A’-A’B’=BA-AB=-(AB-BA)


⇒ (AB-BA)’=-(AB-BA) (skew symmetric matrix)


Eg. Let A =


B=


⇒ AB= and BA=


⇒ AB-BA=


⇒ (AB-BA)’=


⇒ -(AB-BA)=


(ii) BA – 2AB is a Neither Symmetric nor Skew Symmetric matrix


Given A’=A and B’=B


⇒ (BA-2AB)’=(BA)’-(2AB)’


⇒ (BA)’-(2AB)’=A’B’-2B’A’


⇒ A’B’-2B’A’=AB-2BA=-(2BA-AB)


⇒ (BA-2AB)’=-(2BA-AB)


Eg. Let A =


B=


⇒ AB= and BA=


⇒ BA-2AB=



Question 98.

Fill in the blanks in each of the

If A and B are symmetric matrices, then

(i) AB – BA is a _________.

(ii) BA – 2AB is a _________.


Answer:

(i) AB – BA is a Skew Symmetric matrix


Given A’=A and B’=B


⇒ (AB-BA)’=(AB)’-(BA)’


⇒ (AB)’-(BA)’=B’A’-A’B’


⇒ B’A’-A’B’=BA-AB=-(AB-BA)


⇒ (AB-BA)’=-(AB-BA) (skew symmetric matrix)


Eg. Let A =


B=


⇒ AB= and BA=


⇒ AB-BA=


⇒ (AB-BA)’=


⇒ -(AB-BA)=


(ii) BA – 2AB is a Neither Symmetric nor Skew Symmetric matrix


Given A’=A and B’=B


⇒ (BA-2AB)’=(BA)’-(2AB)’


⇒ (BA)’-(2AB)’=A’B’-2B’A’


⇒ A’B’-2B’A’=AB-2BA=-(2BA-AB)


⇒ (BA-2AB)’=-(2BA-AB)


Eg. Let A =


B=


⇒ AB= and BA=


⇒ BA-2AB=



Question 99.

Fill in the blanks in each of the

If A is symmetric matrix, then B’AB is _______.


Answer:

B’AB is a symmetric matrix.

Proof:


Given A is symmetric matrix


⇒ A’=A ..(1)


Now in B’AB,


Let AB=C ..(2)


⇒ B’AB=B’C


Now Using Property (AB)’=B’A’


⇒ (B’C)’=C’(B’)’ (As (B’)’=B)


⇒ C’(B’)’=C’B


⇒ C’B=(AB)’B (Using Property (AB)’=B’A’)


⇒ (AB)’B=B’A’B (Using (1))


⇒ B’A’B= B’AB


⇒ Hence (B’AB)’= B’AB



Question 100.

Fill in the blanks in each of the

If A is symmetric matrix, then B’AB is _______.


Answer:

B’AB is a symmetric matrix.

Proof:


Given A is symmetric matrix


⇒ A’=A ..(1)


Now in B’AB,


Let AB=C ..(2)


⇒ B’AB=B’C


Now Using Property (AB)’=B’A’


⇒ (B’C)’=C’(B’)’ (As (B’)’=B)


⇒ C’(B’)’=C’B


⇒ C’B=(AB)’B (Using Property (AB)’=B’A’)


⇒ (AB)’B=B’A’B (Using (1))


⇒ B’A’B= B’AB


⇒ Hence (B’AB)’= B’AB



Question 101.

Fill in the blanks in each of the

If A and B are symmetric matrices of same order, then AB is symmetric if and only if ______.


Answer:

This is only possible if A and B commute.

Proof:


Given A and B are symmetric matrices,


⇒ A’=A ..(1)


⇒ B’=B ..(2)


Let AB is a Symmetric matrix:-


⇒ (AB)’=AB


Using Property (AB)’=B’A’


⇒ B’A’=AB


⇒ Now using (1) and (2)


⇒ BA=AB


Hence A and B matrix commute.



Question 102.

Fill in the blanks in each of the

If A and B are symmetric matrices of same order, then AB is symmetric if and only if ______.


Answer:

This is only possible if A and B commute.

Proof:


Given A and B are symmetric matrices,


⇒ A’=A ..(1)


⇒ B’=B ..(2)


Let AB is a Symmetric matrix:-


⇒ (AB)’=AB


Using Property (AB)’=B’A’


⇒ B’A’=AB


⇒ Now using (1) and (2)


⇒ BA=AB


Hence A and B matrix commute.



Question 103.

Fill in the blanks in each of the

In applying one or more now operations while finding A–1 by elementary row operations, we obtain all zeros in one or more, then A–1 ______.


Answer:

A-1 Does not exist,


And |A|=0 if there are one or more rows or coloumns with all zero elements.



Question 104.

Fill in the blanks in each of the

In applying one or more now operations while finding A–1 by elementary row operations, we obtain all zeros in one or more, then A–1 ______.


Answer:

A-1 Does not exist,


And |A|=0 if there are one or more rows or coloumns with all zero elements.



Question 105.

Which of the following statements are True or False

A matrix denotes a number.


Answer:

False

A matrix is an ordered rectangular array of numbers of functions.


Only a matrix of order (1×1) denotes a number.


Eg



Question 106.

Which of the following statements are True or False

A matrix denotes a number.


Answer:

False

A matrix is an ordered rectangular array of numbers of functions.


Only a matrix of order (1×1) denotes a number.


Eg



Question 107.

Which of the following statements are True or False

Matrices of any order can be added.


Answer:

False

∵ Matrices having same order can be added.


Eg. Let A =


B=


⇒ A+B=



Question 108.

Which of the following statements are True or False

Matrices of any order can be added.


Answer:

False

∵ Matrices having same order can be added.


Eg. Let A =


B=


⇒ A+B=



Question 109.

Which of the following statements are True or False

Two matrices are equal if they have same number of rows and same number of columns.


Answer:

False

∵ Two matrices are equal if they have same number of rows and same number of columns and corresponding elements within each matrix are equal or identical.


For example:


⇒ A = , B =


Here both matrices have two rows and two columns.


Also, they both have same elements.



Question 110.

Which of the following statements are True or False

Matrices of different order cannot be subtracted.


Answer:

True

∵ Matrices of only same order can be added or subtracted.


Let A =


B=


⇒ A-B= Not possible



Question 111.

Which of the following statements are True or False

Matrices of different order cannot be subtracted.


Answer:

True

∵ Matrices of only same order can be added or subtracted.


Let A =


B=


⇒ A-B= Not possible



Question 112.

Which of the following statements are True or False

Matrix addition is associative as well as commutative.


Answer:

True

1. A+B=B+A (commutative)


2. (A+B)+C= A+(B+C) (associative)



Question 113.

Which of the following statements are True or False

Matrix addition is associative as well as commutative.


Answer:

True

1. A+B=B+A (commutative)


2. (A+B)+C= A+(B+C) (associative)



Question 114.

Which of the following statements are True or False

Matrix multiplication is commutative.


Answer:

False

In general matrix multiplication is not commutative


But it’s associative.


⇒ (AB)C=A(BC)



Question 115.

Which of the following statements are True or False

Matrix multiplication is commutative.


Answer:

False

In general matrix multiplication is not commutative


But it’s associative.


⇒ (AB)C=A(BC)



Question 116.

Which of the following statements are True or False

A square matrix where every element is unity is called an identity matrix.


Answer:

False

∵ A square matrix where every element of the leading diagonal is unity and rest elements are zero is called an identity matrix.


i.e



Question 117.

Which of the following statements are True or False

A square matrix where every element is unity is called an identity matrix.


Answer:

False

∵ A square matrix where every element of the leading diagonal is unity and rest elements are zero is called an identity matrix.


i.e



Question 118.

Which of the following statements are True or False

If A and B are two square matrices of the same order, then A + B = B + A.


Answer:

True

If A and B are two square matrices of the same order, then A + B = B + A ( Property of square matrix)


Eg





Question 119.

Which of the following statements are True or False

If A and B are two square matrices of the same order, then A + B = B + A.


Answer:

True

If A and B are two square matrices of the same order, then A + B = B + A ( Property of square matrix)


Eg





Question 120.

Which of the following statements are True or False

If A and B are two matrices of the same order, then A – B = B – A.


Answer:

False

∵ If A and B are two matrices of the same order,


then A – B = -(B – A)


Eg






Question 121.

Which of the following statements are True or False

If A and B are two matrices of the same order, then A – B = B – A.


Answer:

False

∵ If A and B are two matrices of the same order,


then A – B = -(B – A)


Eg






Question 122.

Which of the following statements are True or False

If matrix AB = O, then A = O or B = O or both A and B are null matrices.


Answer:

False

∵ Its not necessary that for multiplication of matrix A and B to be 0 one of them has to be a null matrix.


Eg




Question 123.

Which of the following statements are True or False

If matrix AB = O, then A = O or B = O or both A and B are null matrices.


Answer:

False

∵ Its not necessary that for multiplication of matrix A and B to be 0 one of them has to be a null matrix.


Eg




Question 124.

Which of the following statements are True or False

Transpose of a column matrix is a column matrix.


Answer:

False

∵ Transpose of a column matrix is a Row matrix and vice-versa.


Let A=


⇒ A’=



Question 125.

Which of the following statements are True or False

Transpose of a column matrix is a column matrix.


Answer:

False

∵ Transpose of a column matrix is a Row matrix and vice-versa.


Let A=


⇒ A’=



Question 126.

Which of the following statements are True or False

If A and B are two square matrices of the same order, then AB = BA.


Answer:

False

∵ Matrix multiplication is not commutative.


Eg. Let A =


B=


⇒ AB= and BA=


⇒ AB≠BA



Question 127.

Which of the following statements are True or False

If A and B are two square matrices of the same order, then AB = BA.


Answer:

False

∵ Matrix multiplication is not commutative.


Eg. Let A =


B=


⇒ AB= and BA=


⇒ AB≠BA



Question 128.

Which of the following statements are True or False

If each of the three matrices of the same order are symmetric, then their sum is a symmetric matrix.


Answer:

True

Eg






Question 129.

Which of the following statements are True or False

If A and B are any two matrices of the same order, then (AB)’ = A’B’.


Answer:

False

∵If A and B are any two matrices for which AB is defined, then


(AB)’=B’A’.



Question 130.

Which of the following statements are True or False

If A and B are any two matrices of the same order, then (AB)’ = A’B’.


Answer:

False

∵If A and B are any two matrices for which AB is defined, then


(AB)’=B’A’.



Question 131.

Which of the following statements are True or False

If (AB)’ = B’A’, where A and B are not square matrices, then number of rows in A is equal to number of columns in B and number of columns in A is equal to number of rows in B.


Answer:

True

∵If A and B are any two matrices for which AB is defined, then


(AB)’=B’A’.



Question 132.

Which of the following statements are True or False

If (AB)’ = B’A’, where A and B are not square matrices, then number of rows in A is equal to number of columns in B and number of columns in A is equal to number of rows in B.


Answer:

True

∵If A and B are any two matrices for which AB is defined, then


(AB)’=B’A’.



Question 133.

Which of the following statements are True or False

If A, B and C are square matrices of same order, then AB = AC always implies that B = C.


Answer:

False

∵ If AB = AC => B=C


The above condition is only possible if matrix A is invertible


(i.e |A|≠0).


⇒ If A is invertible, then


⇒ A-1(AB)= A-1(AC)


⇒ (A-1A)B = (A-1A)C


⇒ IB=IC


⇒ B=C



Question 134.

Which of the following statements are True or False

If A, B and C are square matrices of same order, then AB = AC always implies that B = C.


Answer:

False

∵ If AB = AC => B=C


The above condition is only possible if matrix A is invertible


(i.e |A|≠0).


⇒ If A is invertible, then


⇒ A-1(AB)= A-1(AC)


⇒ (A-1A)B = (A-1A)C


⇒ IB=IC


⇒ B=C



Question 135.

Which of the following statements are True or False

AA’ is always a symmetric matrix for any square matrix A.


Answer:

True

(AA’)’=(A’)’A’


As we know (A) = A


(AA’)’=AA’ (Condition of symmetric matrix)



Question 136.

Which of the following statements are True or False

AA’ is always a symmetric matrix for any square matrix A.


Answer:

True

(AA’)’=(A’)’A’


As we know (A) = A


(AA’)’=AA’ (Condition of symmetric matrix)



Question 137.

Which of the following statements are True or False

If then AB and BA are defined and equal.


Answer:

False

∵ Here A has an order (2×3) and B has an order (3×2),


Hence AB is defined and will give an output matrix of order (2×2)


And BA is also defined but will give an output matrix of order (3×3).


⇒ AB ≠ BA



Question 138.

Which of the following statements are True or False

If then AB and BA are defined and equal.


Answer:

False

∵ Here A has an order (2×3) and B has an order (3×2),


Hence AB is defined and will give an output matrix of order (2×2)


And BA is also defined but will give an output matrix of order (3×3).


⇒ AB ≠ BA



Question 139.

Which of the following statements are True or False

If A is skew symmetric matrix, then A2 is a symmetric matrix.


Answer:

True

For skew symmetric matrix A’=-A


⇒ (A2)’=(AA)’=A’A’


⇒ A’A’=(-A)(-A)=A2


⇒ (A2)’=A2 (symmetric matrix)



Question 140.

Which of the following statements are True or False

If A is skew symmetric matrix, then A2 is a symmetric matrix.


Answer:

True

For skew symmetric matrix A’=-A


⇒ (A2)’=(AA)’=A’A’


⇒ A’A’=(-A)(-A)=A2


⇒ (A2)’=A2 (symmetric matrix)



Question 141.

Which of the following statements are True or False

(AB)–1 = A–1.B–1, where A and B are invertible matrices satisfying cumulative property with respect to multiplication.


Answer:

False

∵ If A and B are invertible matrices then,


⇒ (AB)-1=B-1A-1



Question 142.

Which of the following statements are True or False

(AB)–1 = A–1.B–1, where A and B are invertible matrices satisfying cumulative property with respect to multiplication.


Answer:

False

∵ If A and B are invertible matrices then,


⇒ (AB)-1=B-1A-1